Docsity
Docsity

Prepare for your exams
Prepare for your exams

Study with the several resources on Docsity


Earn points to download
Earn points to download

Earn points by helping other students or get them with a premium plan


Guidelines and tips
Guidelines and tips

Fisdap Cardiology Review (Paramedic) 2024-2025 Version. Questions & Answers. Graded A+, Exams of Nursing

Fisdap Cardiology Review (Paramedic) 2024-2025 Version. Real Questions & Answers-(100% Correct) Already Graded A+

Typology: Exams

2023/2024

Available from 06/11/2024

zaza-maica
zaza-maica 🇬🇧

319 documents

1 / 120

Toggle sidebar

Partial preview of the text

Download Fisdap Cardiology Review (Paramedic) 2024-2025 Version. Questions & Answers. Graded A+ and more Exams Nursing in PDF only on Docsity! Fisdap Cardiology Review (Paramedic) 2024-2025 Version. Real Questions & Answers-(100% Correct) Already Graded A+ 3 signs of Becks triad - ANSElevated pulse pressure, muffled heart tones, and hypotension 3rd trimester bright red and painless vaginal bleeding - ANSPlacenta previa 3rd trimester dark red vaginal bleeding with severe pain - ANSAbruptio placenta A 145-pound man requires a dopamine infusion at 15 µg/kg/min for severe hypotension. You have a premixed bag containing 800 mg of dopamine in 500 mL of normal saline. If you are using a microdrip administration set (60 gtts/mL), how many drops per minute should you deliver to achieve the required dose? A: 42 B: 36 C: 48 D: 30 - ANS*B: 36* Reason: First, convert the patient's weight from pounds to kilograms: 145 ÷ 2.2 = 66 kg. Next, determine the desired dose: 15 µg/kg/min × 66 kg = 990 µg/min. The next step is to determine the concentration of dopamine on hand: 800 mg ÷ 500 mL = 1.6 mg/mL (1,600 µg/mL [1.6 × 1,000 = 1,600]). Now, you must determine the number of mL to be delivered per minute: 990 µg/min [desired dose] ÷ 1,600 µg/mL [concentration on hand] = 0.6 mL/min. The final step is to determine the number of drops per minute that you must set your IV flow rate at: 0.6 mL/min × 60 gtts/mL (drop factor of the microdrip) ÷ 1 (total infusion time in minutes) = 36 gtts/min. A 145-pound man requires a dopamine infusion at 15 µg/kg/min for severe hypotension. You have a premixed bag containing 800 mg of dopamine in 500 mL of normal saline. If you are using a microdrip administration set (60 gtts/mL), how many drops per minute should you deliver to achieve the required dose? A: 48 B: 42 C: 30 D: 36 - ANS*D: 36* A 145-pound man requires a dopamine infusion at 15 µg/kg/min for severe hypotension. You have a premixed bag containing 800 mg of dopamine in 500 mL of normal saline. If you are using a microdrip administration set (60 gtts/mL), how many drops per minute should you deliver to achieve the required dose? - ANS36. First, convert the patient's weight from pounds to kilograms: 145 ÷ 2.2 = 66 kg. Next, determine the desired dose: 15 µg/kg/min × 66 kg = 990 µg/min. The next step is to determine the concentration of dopamine on hand: 800 mg ÷ 500 mL = 1.6 mg/mL (1,600 µg/mL [1.6 × 1,000 = 1,600]). Now, you must determine the number of mL to be delivered per minute: 990 µg/min [desired dose] ÷ 1,600 µg/mL [concentration on hand] = 0.6 mL/min. The final step is to determine the number of drops per minute that you must set your IV flow rate at: 0.6 mL/min × 60 gtts/mL (drop factor of the microdrip) ÷ 1 (total infusion time in minutes) = 36 gtts/min. A 27-year-old female complains of palpitations. The cardiac monitor reveals a narrow- complex tachycardia at 180/min. She denies any other symptoms, and states that this has happened to her before, but it typically resolves on its own. Her blood pressure is 126/66 mm Hg, pulse is 180 beats/min, and respirations are 16 breaths/min. After attempting vagal maneuvers and giving two doses of adenosine, her cardiac rhythm and vital signs remain unchanged. You should: A: infuse 150 mg of amiodarone over 10 minutes, reassess her, and repeat the amiodarone if needed. B: transport at once, reassess her frequently, and perform synchronized cardioversion if necessary. C: administer 5 mg of midazolam and perform synchronized cardioversion starting with 50 joules. D: administer 0.35 mg/kg of diltiazem over 2 minutes and then reassess her hemodynamic status. - ANS*B: transport at once, reassess her frequently, and perform synchronized cardioversion if necessary.* Reason: Although the patient is in supraventricular tachycardia (SVT), she remains stable following your initial efforts to slow her heart rate with vagal maneuvers and adenosine. Her failure to respond to initial treatment does not automatically make her unstable. Simply transport her, closely monitor her en route, and be prepared to cardiovert her if she does become unstable (ie, hypotension, altered mental status, chest pain). Unless specified in your local protocols, pharmacologic therapy beyond adenosine (ie, calcium channel blockers, amiodarone) is typically not indicated in the field for stable patients with SVT, although these medications may be given in the emergency department. However, if your protocols or medical control call for the administration of diltiazem (Cardizem), the initial dose is 0.25 mg/kg. A 27-year-old female complains of palpitations. The cardiac monitor reveals a narrow- complex tachycardia at 180/min. She denies any other symptoms, and states that this has happened to her before, but it typically resolves on its own. Her blood pressure is 126/66 mm Hg, pulse is 180 beats/min, and respirations are 16 breaths/min. After frequent premature atrial complexes (PACs), which indicates atrial irritability, suggests paroxysmal supraventricular tachycardia (PSVT) as the underlying dysrhythmia that caused her syncopal episode. In PSVT, the heart is beating so fast that ventricular filling and cardiac output decrease, which results in a transient decrease in cerebral perfusion. Not all patients with PSVT experience syncope. Many experience an acute onset of palpitations and/or lightheadedness that spontaneously resolves. A 35-year-old female experienced a syncopal episode shortly after complaining of palpitations. She was reportedly unconscious for less than 10 seconds. Upon your arrival, she is conscious and alert, denies any injuries, and states that she feels fine. She further denies any significant medical history. Her vital signs are stable and the cardiac monitor reveals a sinus rhythm with frequent premature atrial complexes. On the basis of this information, what MOST likely caused her syncopal episode? A: A brief episode of ventricular tachycardia B: Aberrant conduction through the ventricles C: Paroxysmal supraventricular tachycardia D: A sudden increase in cardiac output - ANS*C: Paroxysmal supraventricular tachycardia* Reason: Syncope (fainting) of cardiac origin is caused by a sudden decrease in cerebral perfusion secondary to a decrease in cardiac output. This is usually the result of an acute bradydysrhythmia or tachydysrhythmia. In this particular patient, the presence of frequent premature atrial complexes (PACs), which indicates atrial irritability, suggests paroxysmal supraventricular tachycardia (PSVT) as the underlying dysrhythmia that caused her syncopal episode. In PSVT, the heart is beating so fast that ventricular filling and cardiac output decrease, which results in a transient decrease in cerebral perfusion. Not all patients with PSVT experience syncope. Many experience an acute onset of palpitations and/or lightheadedness that spontaneously resolves. A 35-year-old female experienced a syncopal episode shortly after complaining of palpitations. She was reportedly unconscious for less than 10 seconds. Upon your arrival, she is conscious and alert, denies any injuries, and states that she feels fine. She further denies any significant medical history. Her vital signs are stable and the cardiac monitor reveals a sinus rhythm with frequent premature atrial complexes. On the basis of this information, what MOST likely caused her syncopal episode? - ANSParoxysmal supraventricular tachycardia. Syncope (fainting) of cardiac origin is caused by a sudden decrease in cerebral perfusion secondary to a decrease in cardiac output. This is usually the result of an acute bradydysrhythmia or tachydysrhythmia. In this particular patient, the presence of frequent premature atrial complexes (PACs), which indicates atrial irritability, suggests paroxysmal supraventricular tachycardia (PSVT) as the underlying dysrhythmia that caused her syncopal episode. In PSVT, the heart is beating so fast that ventricular filling and cardiac output decrease, which results in a transient decrease in cerebral perfusion. Not all patients with PSVT experience syncope. Many experience an acute onset of palpitations and/or lightheadedness that spontaneously resolves. A 38-year-old woman who is experiencing abnormal weight gain in her neck/upper back and growth of facial hair could possibly have which condition? - ANSCushing's syndrome A 39-year-old female presents with an acute onset of lightheadedness. The cardiac monitor reveals a tachycardic rhythm at 170 beats/min with QRS complexes that measure 0.08 seconds in duration. Despite vagal maneuvers and adenosine, her cardiac rhythm remains unchanged. She is conscious and alert, has a blood pressure of 118/72 mm Hg, and denies shortness of breath or chest discomfort. You should: A: administer 150 mg of amiodarone over 10 minutes. B: consider that her rhythm is ventricular in origin. C: perform synchronized cardioversion with 50 joules. D: transport immediately and monitor her en route. - ANS*D: transport immediately and monitor her en route* Reason: The patient in this scenario is in supraventricular tachycardia (SVT); her heart rate is 170 beats/min and her QRS complexes are narrow (< 0.12 seconds). Despite appropriate treatment for her rhythm (ie, vagal maneuvers, adenosine), her rhythm remains unchanged, although she remains hemodynamically stable. Lightheadedness is common in patients with SVT, but it is not a clinical indicator of hemodynamic instability. A cardiac rhythm of ventricular origin (eg, ventricular tachycardia) is characterized by QRS complexes that are greater than 0.12 seconds in duration; this patient's QRS complexes are 0.08 seconds in duration. If vagal maneuvers and adenosine are unsuccessful in converting her rhythm, transport immediately without further treatment (other than oxygen); her present condition indicates that she is tolerating the cardiac rhythm. However, if signs of hemodynamic instability are noted (ie, hypotension, decreased level of consciousness, chest pain, shortness of breath), perform synchronized cardioversion at 50 to 100 joules without delay. A 39-year-old female presents with an acute onset of lightheadedness. The cardiac monitor reveals a tachycardic rhythm at 170 beats/min with QRS complexes that measure 0.08 seconds in duration. Despite vagal maneuvers and adenosine, her cardiac rhythm remains unchanged. She is conscious and alert, has a blood pressure of 118/72 mm Hg, and denies shortness of breath or chest discomfort. You should: - ANSTransport immediately and monitor her en route. The patient in this scenario is in supraventricular tachycardia (SVT); her heart rate is 170 beats/min and her QRS complexes are narrow (< 0.12 seconds). Despite appropriate treatment for her rhythm (ie, vagal maneuvers, adenosine), her rhythm remains unchanged, although she remains hemodynamically stable. Lightheadedness is common in patients with SVT, but it is not a clinical indicator of hemodynamic instability. A cardiac rhythm of ventricular origin (eg, ventricular tachycardia) is characterized by QRS complexes that are greater than 0.12 seconds in duration; this patient's QRS complexes are 0.08 seconds in duration. If vagal maneuvers and adenosine are unsuccessful in converting her rhythm, transport immediately without further treatment (other than oxygen); her present condition indicates that she is tolerating the cardiac rhythm. However, if signs of hemodynamic instability are noted (ie, hypotension, decreased level of consciousness, chest pain, shortness of breath), perform synchronized cardioversion at 50 to 100 joules without delay. A 44-year-old man presents with the rhythm shown below. He complains of nausea, but denies vomiting. He is conscious and alert with a BP of 122/62 mm Hg, a pulse rate of 98 beats/min, and respirations of 16 breaths/min and unlabored. Treatment for this patient would MOST likely include: - ANSOndansetron, 4 mg. Unless associated with a fast rate (> 100 beats/min) and hemodynamic compromise (eg, hypotension, altered mental status, pulmonary edema), treatment for atrial flutter is usually not necessary in the prehospital setting. Administer supplemental oxygen if indicated, transport, and monitor the patient's hemodynamic status en route. For this patient, you should treat his nausea with an antiemetic, such as ondansetron (Zofran), 4 mg; or promethazine (Phenergan), 12.5 to 25 mg. A 47-year-old male took two of his prescribed nitroglycerin tablets prior to calling EMS. When you arrive at the scene, the patient tells you that he has a throbbing headache and is still experiencing chest pain. Your MOST immediate suspicion should be that: A: his chest pain is probably not of a cardiac origin. B: permanent myocardial damage has already occurred. C: he is experiencing continued myocardial ischemia. D: his nitroglycerin is outdated or has lost its potency - ANS*C: he is experiencing continued myocardial ischemia* Reason: When a patient reports taking nitroglycerin (NTG) for chest pain, you should determine how many tablets or sprays he or she took, and whether or not the NTG relieved his or her pain. Failure of NTG to relieve cardiac-related chest pain can occur for one of two reasons—the pain is of extraordinary severity, such as that associated with acute myocardial infarction, or the NTG has been open too long and has lost its potency. Fresh, potent NTG has certain distinct side effects, including a throbbing headache, a burning sensation under the tongue, and a bitter taste. If the patent did not experience any of these side effects, chances are the drug was outdated or had lost its potency. However, if the patient experienced any of these side effects, but is still experiencing chest pain, you should suspect that he or she is experiencing continued myocardial ischemia and is in the process of having an acute myocardial infarction. A 12-lead ECG and other diagnostic tests (ie, echocardiography) are required to determine if permanent myocardial damage has occurred. A 47-year-old male took two of his prescribed nitroglycerin tablets prior to calling EMS. When you arrive at the scene, the patient tells you that he has a throbbing headache and is still experiencing chest pain. Your MOST immediate suspicion should be that: - ANSHe is experiencing continued myocardial ischemia. A 50-year-old woman is pulseless and apneic. Your partner and an emergency medical responder are performing well-coordinated CPR. After 2 minutes of CPR, the cardiac monitor reveals coarse ventricular fibrillation. You should: A: shock the patient three times with 360 monophasic joules. B: deliver a single shock and immediately resume CPR. C: defibrillate at once and then reassess the rhythm and pulse. D: assess for a carotid pulse for no longer than 10 seconds. - ANS*B: deliver a single shock and immediately resume CPR.* Reason: A single shock (360 monophasic joules or the biphasic equivalent) should be administered to the patient with V-Fib or pulseless V-Tach cardiac arrest. Immediately following this single shock, begin or resume CPR, starting with chest compressions. Assessing the patient's cardiac rhythm and pulse immediately following defibrillation causes an unnecessary delay in CPR, and delays in CPR have been directly linked to poor patient outcomes. Most patients who are defibrillated—especially if their arrest interval is prolonged—remain in V-Fib/pulseless V-Tach or convert to another non- perfusing rhythm (ie, asystole, PEA). Either way, the patient is still in cardiac arrest and needs immediate CPR. After 2 minutes of CPR, reassess the patient's rhythm, and if necessary, a pulse (if an organized cardiac rhythm appears), and repeat defibrillation (single shock) if indicated, followed immediately by CPR. A 50-year-old woman is pulseless and apneic. Your partner and an emergency medical responder are performing well-coordinated CPR. After 2 minutes of CPR, the cardiac monitor reveals coarse ventricular fibrillation. You should: - ANSDeliver a single shock and immediately resume CPR. A single shock (360 monophasic joules or the biphasic equivalent) should be administered to the patient with V-Fib or pulseless V-Tach cardiac arrest. Immediately following this single shock, begin or resume CPR, starting with chest compressions. Assessing the patient's cardiac rhythm and pulse immediately following defibrillation causes an unnecessary delay in CPR, and delays in CPR have been directly linked to poor patient outcomes. Most patients who are defibrillated—especially if their arrest interval is prolonged—remain in V-Fib/pulseless V-Tach or convert to another non- perfusing rhythm (ie, asystole, PEA). Either way, the patient is still in cardiac arrest and needs immediate CPR. After 2 minutes of CPR, reassess the patient's rhythm, and if necessary, a pulse (if an organized cardiac rhythm appears), and repeat defibrillation (single shock) if indicated, followed immediately by CPR. A 54-year-old man presents with chest pressure, confusion, and profuse diaphoresis. As your partner administers supplemental oxygen, you apply the cardiac monitor. In lead II, you observe a wide QRS complex rhythm with dissociated P waves and a ventricular rate of 35 beats/min. You should: A: immediately obtain a 12-lead ECG. B: start an IV and give 0.5 mg of atropine. C: begin transcutaneous pacing at once. D: obtain a complete set of vital signs. - ANS*C: begin transcutaneous pacing at once* Reason: The patient in this scenario is in a third-degree (complete) AV block, which is causing his signs and symptoms. Complete heart block should be treated with immediate transcutaneous cardiac pacing (TCP). Given the patient's clinical presentation, it is clear that he is hemodynamically unstable; obtaining a complete set of vital signs will yield very little, if any, additional information. A 12-lead ECG should be obtained, but not before addressing the most immediate problem of hemodynamic compromise. Atropine should be avoided in patients with high-grade AV heart blocks (eg, second-degree AV block type II and third-degree AV block). Atropine may worsen the patient's condition— especially in cases of third-degree AV block—by increasing sinus node discharge without any effect on the ventricles. Remember, if the rhythm is perfusing, but is slow and wide, begin TCP without delay. A 54-year-old man presents with chest pressure, confusion, and profuse diaphoresis. As your partner administers supplemental oxygen, you apply the cardiac monitor. In lead II, you observe a wide QRS complex rhythm with dissociated P waves and a ventricular rate of 35 beats/min. You should: - ANSBegin transcutaneous pacing at once. The patient in this scenario is in a third-degree (complete) AV block, which is causing his signs and symptoms. Complete heart block should be treated with immediate transcutaneous cardiac pacing (TCP). Given the patient's clinical presentation, it is clear that he is hemodynamically unstable; obtaining a complete set of vital signs will yield very little, if any, additional information. A 12-lead ECG should be obtained, but not before addressing the most immediate problem of hemodynamic compromise. Atropine should be avoided in patients with high-grade AV heart blocks (eg, second-degree AV block type II and third-degree AV block). Atropine may worsen the patient's condition— especially in cases of third-degree AV block—by increasing sinus node discharge without any effect on the ventricles. Remember, if the rhythm is perfusing, but is slow and wide, begin TCP without delay. A 56-year-old man complains of chest discomfort, shortness of breath, and is profusely diaphoretic. His blood pressure is 84/64 mm Hg and his radial pulses are barely palpable. You should: A: consider sedation and perform cardioversion. B: defibrillate with 200 biphasic joules. C: prepare for immediate cardiac pacing. D: give 150 mg of amiodarone over 10 minutes. - ANS*A: consider sedation and perform cardioversion* Reason: Your patient has a narrow-complex tachycardia, probably supraventricular tachycardia (SVT). Furthermore, he is hemodynamically unstable as evidenced by his hypotension, respiratory distress, and chest discomfort. Heart rates greater than 150 beats/min often cause hemodynamic compromise because they impair ventricular filling and subsequent cardiac output. Patients with unstable tachycardias require synchronized cardioversion. For the patient with a regular narrow-complex tachycardia (ie, SVT), start with 50 to 100 joules. Consider sedating the patient prior to cardioversion only if doing so does not delay the procedure. If the initial cardioversion attempt is unsuccessful, repeat the cardioversion, increasing the energy setting in a stepwise fashion, and search for potentially reversible underlying causes. Defibrillation is indicated for patients with V-Fib and pulseless V-Tach. Transcutaneous cardiac pacing (TCP) is indicated for patients with hemodynamically unstable bradycardia. Amiodarone, in a dose of 150 mg over 10 minutes, is indicated for patients with stable narrow or wide-complex tachycardias. A 56-year-old man has had chest pain for the past 2 days, but refused to go to the hospital. His wife called EMS when she noticed that he was not acting right. He is conscious, but confused, and is diaphoretic. His BP is 80/40 mm Hg and his pulse is rapid and weak. The patient's history and your assessment findings are MOST consistent with: A: unstable angina pectoris. B: cardiogenic hypoperfusion. C: acute myocardial infarction. D: acute ischemic stroke. - ANS*B: cardiogenic hypoperfusion.* Reason: The patient most likely experienced an acute myocardial infarction (AMI); however, since he did not receive timely treatment, extensive myocardial damage has resulted in pump failure. His low BP; weak, rapid pulses; and altered mental status indicate that he is systemically hypoperfused. Hypoperfusion (shock) secondary to a cardiac etiology (ie, pump failure, fast or slow heart rate) is called cardiogenic shock. True cardiogenic shock, which occurs when the myocardium is extensively and permanently damaged and can no longer meet the metabolic needs of the body, has a high mortality rate. A 56-year-old man has had chest pain for the past 2 days, but refused to go to the hospital. His wife called EMS when she noticed that he was not acting right. He is conscious, but confused, and is diaphoretic. His BP is 80/40 mm Hg and his pulse is rapid and weak. The patient's history and your assessment findings are MOST consistent with: - ANSCardiogenic hypoperfusion. The patient most likely experienced an acute myocardial infarction (AMI); however, since he did not receive timely treatment, extensive myocardial damage has resulted in pump failure. His low BP; weak, rapid pulses; and altered mental status indicate that he is systemically hypoperfused. Hypoperfusion (shock) secondary to a cardiac etiology (ie, pump failure, fast or slow heart rate) is called cardiogenic shock. True cardiogenic C: give her up to 3 sublingual doses of nitroglycerin. D: administer 12 mg of adenosine rapid IV push. - ANS*B: have her chew and swallow 325 mg of aspirin* Reason: A regular, narrow complex tachycardia at a rate greater than 150 beats/min is consistent with supraventricular tachycardia (SVT). Although the patient is conscious and alert, she is complaining of chest discomfort and is hypotensive. Since she could be experiencing an acute coronary syndrome (ACS), you should instruct her to chew and swallow up to 325 mg of aspirin. Aspirin should be given to any patient suspected of experiencing an ACS, provided there are no contraindications (eg, allergy); it will not affect her blood pressure. Nitroglycerin, however, may exacerbate her hypotension and should be avoided. You can consider administering adenosine; however, the initial dose is 6 mg rapid IV push. Amiodarone, in a dose of 150 mg over 10 minutes, is appropriate for patients with hemodynamically stable wide-complex tachycardias (ie, V-Tach). Closely monitor this patient and be prepared to perform synchronized cardioversion. A 59-year-old woman presents with a regular, narrow-complex tachycardia at a rate of 180 beats/min. She is conscious and alert, but complains of chest discomfort and has a blood pressure of 86/56 mm Hg. In addition to giving her supplemental oxygen, you should: - ANSHave her chew and swallow 325 mg of aspirin. A regular, narrow complex tachycardia at a rate greater than 150 beats/min is consistent with supraventricular tachycardia (SVT). Although the patient is conscious and alert, she is complaining of chest discomfort and is hypotensive. Since she could be experiencing an acute coronary syndrome (ACS), you should instruct her to chew and swallow up to 325 mg of aspirin. Aspirin should be given to any patient suspected of experiencing an ACS, provided there are no contraindications (eg, allergy); it will not affect her blood pressure. Nitroglycerin, however, may exacerbate her hypotension and should be avoided. You can consider administering adenosine; however, the initial dose is 6 mg rapid IV push. Amiodarone, in a dose of 150 mg over 10 minutes, is appropriate for patients with hemodynamically stable wide-complex tachycardias (ie, V-Tach). Closely monitor this patient and be prepared to perform synchronized cardioversion. A 60-year-old female presents with confusion, shortness of breath, and diaphoresis. Her blood pressure is 70/40 mm Hg and her heart rate is 40 beats/min. The cardiac monitor reveals a slow, wide complex rhythm with dissociated P waves. After applying supplemental oxygen, you should: A: give her up to 325 mg of baby aspirin. B: begin immediate transcutaneous pacing. C: start an IV and give a rapid fluid bolus. D: start an IV and give 0.5 mg of atropine. - ANS*B: begin immediate transcutaneous pacing* A 60-year-old female presents with confusion, shortness of breath, and diaphoresis. Her blood pressure is 70/40 mm Hg and her heart rate is 40 beats/min. The cardiac monitor reveals a slow, wide complex rhythm with dissociated P waves. After applying supplemental oxygen, you should: - ANSBegin immediate transcutaneous pacing. The cardiac rhythm described is a third-degree (complete) AV block, and the patient is clinically unstable (ie, hypotension, altered mental status, shortness of breath). Third- degree AV block is characterized by a slow ventricular rate and no P-to-QRS relationship (AV dissociation). Patients with high-grade AV blocks (eg, second-degree type II, third-degree) are often clinically unstable and require immediate transcutaneous cardiac pacing (TCP). Atropine is an appropriate drug for clinically unstable patients with sinus bradycardia and bradycardia associated with low-grade AV blocks (eg, first- degree, second-degree type I); it is not recommended for high-grade AV blocks. If TCP is unsuccessful for this patient, consider an epinephrine infusion (2 to 10 µg/min) or a dopamine infusion (5 to 10 µg/kg/min), either of which may increase her heart rate and blood pressure. The patient's hypotension is secondary to severe bradycardia, not hypovolemia; therefore, a rapid IV fluid bolus is not indicated. If you have reason to suspect that the patient is experiencing an acute coronary syndrome (ACS), aspirin should be given. A 60-year-old man presents with chest discomfort, diaphoresis, and dyspnea. The 12- lead ECG reveals 4-mm ST segment elevation in leads V1 through V4. You should suspect: A: anterolateral infarct. B: inferoseptal ischemia. C: anterolateral injury. D: anteroseptal injury. - ANS*D: anteroseptal injury.* Reason: The precordial (chest) leads view the following aspects of the heart: V1 and V2, interventricular septum; V3 and V4, anterior wall; V5 and V6, lateral wall. ST segment depression and/or T wave inversion in two or more contiguous leads indicates ischemia. ST segment that is equal to or greater than 1-mm in two or more contiguous leads indicates injury. A developing Q wave may be seen in conjunction with ST segment elevation associated with myocardial injury. Therefore, 4-mm ST segment elevation in leads V1 through V4 indicates an anteroseptal injury pattern (acute MI in progress). Infarcted (dead [necrotic]) myocardium is characterized by poor R wave progression in the precordial leads and/or the presence of a pathologic Q wave in two or more contiguous leads. By definition, a pathologic Q wave is wider than 0.04 seconds (40 ms) or deeper than one third the height of the R wave that follows it. A 60-year-old man presents with chest discomfort, diaphoresis, and dyspnea. The 12- lead ECG reveals 4-mm ST segment elevation in leads V1 through V4. You should suspect: - ANSAnteroseptal injury. The precordial (chest) leads view the following aspects of the heart: V1 and V2, interventricular septum; V3 and V4, anterior wall; V5 and V6, lateral wall. ST segment depression and/or T wave inversion in two or more contiguous leads indicates ischemia. ST segment that is equal to or greater than 1-mm in two or more contiguous leads indicates injury. A developing Q wave may be seen in conjunction with ST segment elevation associated with myocardial injury. Therefore, 4-mm ST segment elevation in leads V1 through V4 indicates an anteroseptal injury pattern (acute MI in progress). Infarcted (dead [necrotic]) myocardium is characterized by poor R wave progression in the precordial leads and/or the presence of a pathologic Q wave in two or more contiguous leads. By definition, a pathologic Q wave is wider than 0.04 seconds (40 ms) or deeper than one third the height of the R wave that follows it. A 61-year-old male presents with chest pressure that woke him up from his nap 30 minutes ago. He is diaphoretic, anxious, and rates his pain as an an 8 over 10. His past medical history is significant for hypertension, type II diabetes, and coronary stent placement 2 months ago. He takes lisinopril, Plavix, and Glucophage, and is wearing a medical alert bracelet stating "allergic to salicylates." His blood pressure is 160/100 mm Hg, pulse is 110 beats/min, and respirations are 22 breaths/min. The 12-lead ECG shows sinus tachycardia with 3-mm ST segment elevation in leads V1 through V5. Which of the following treatment modalities is MOST appropriate for this patient? A: 325 mg of baby aspirin, supplemental oxygen, vascular access, up to three doses of nitroglycerin, and up to 10 mg of morphine if his systolic BP is greater than 120 mm Hg and he is still in pain B: 325 mg of baby aspirin; high-flow oxygen via nonrebre - ANS*D: Supplemental oxygen, vascular access, up to three 0.4 mg doses of nitroglycerin, and 2 to 4 mg of morphine sulfate if his systolic BP is greater than 90 mm Hg and he is still experiencing pain* Reason: The patient is experiencing an acute coronary syndrome (ACS). His 12-lead ECG indicates anteroseptal injury with lateral extension (ST elevation in leads V1 through V5). Appropriate treatment includes oxygen (maintain an SpO2 of greater than 94%), vascular access, up to three 0.4 mg doses of nitroglycerin (NTG), and 2 to 4 mg of morphine if NTG fails to relieve his pain and his systolic BP is above 90 mm Hg. Some EMS systems may use fentanyl (Sublimaze) for analgesia. Aspirin, a salicylate, is also given to patients with ACS; however, this patient is allergic to salicylates. Obtain a right- sided 12-lead ECG in patients with signs of inferior wall injury (ST elevation in leads II, III, aVF). Inferior wall infarctions may involve the right ventricle; a right-sided 12-lead ECG will help confirm this. Apply the multi-pads to the patient, not because he is at risk for bradycardia (more common with inferior infarctions), but because he is at risk for cardiac arrest due to V-Fib or pulseless V-Tach. A 61-year-old male presents with chest pressure that woke him up from his nap 30 minutes ago. He is diaphoretic, anxious, and rates his pain as an an 8 over 10. His past medical history is significant for hypertension, type II diabetes, and coronary stent placement 2 months ago. He takes lisinopril, Plavix, and Glucophage, and is wearing a medical alert bracelet stating "allergic to salicylates." His blood pressure is 160/100 mm Hg, pulse is 110 beats/min, and respirations are 22 breaths/min. The 12-lead ECG shows sinus tachycardia with 3-mm ST segment elevation in leads V1 through V5. This patient's clinical presentation and his history of hypertension and transient ischemic attacks (TIAs) suggest acute ischemic stroke. However, his blood glucose level (BGL) is significantly low and must be treated. Untreated hypoglycemia may cause irreversible brain damage or death. Appropriate treatment for this patient involves administering 50% dextrose (consider giving 12.5 g) and then reassessing his BGL to determine the need for additional glucose. Because the patient is confused, and because some patients with acute ischemic stroke lose protective airway reflexes, oral glucose should be avoided. He may not be able to swallow it, which may result in aspiration. Further treatment includes protecting his impaired extremities from injury, monitoring his cardiac rhythm, and transporting him to the hospital. Notify the receiving facility early. Aspirin should be avoided in the prehospital setting for patients with signs and symptoms of a stroke. A CT scan of the head must be performed first to rule out intracranial hemorrhage. A 70-year-old man presents with the cardiac rhythm shown below. He is confused, is slow to answer your questions, and is profusely diaphoretic. His blood pressure is 76/54 mm Hg, his pulse is rapid and weak, and his respirations are 22 breaths/min and labored. He is receiving high-flow oxygen and your partner has established a patent IV line. You should: - ANSConsider sedation and then cardiovert with 100 joules. This patient is in ventricular tachycardia (V-Tach). Furthermore, he is hemodynamically unstable as evidenced by his confusion, hypotension, and labored breathing. Therefore, he requires prompt synchronized cardioversion, starting with 100 joules. Consider sedation with midazolam (Versed) or diazepam (Valium), but do not allow this to delay cardioversion. Amiodarone would be an appropriate intervention if the patient was hemodynamically stable. Vagal maneuvers and adenosine are appropriate for stable patients with narrow complex tachycardias (eg, SVT). Fluid boluses will likely not improve the patient's blood pressure; his hypotension is the result of inadequate ventricular filling and decreased cardiac output due to his cardiac rhythm—not hypovolemia. A 70-year-old woman was suddenly awakened with the feeling that she was suffocating. She is anxious, is laboring to breathe, and has dried blood on her lips. The ECG shows the cardiac rhythm below. Which of the following pathophysiologies BEST explains her clinical presentation? - ANSDecreased stroke volume with left heart failure. Paroxysmal nocturnal dyspnea (PND), the sudden awakening from sleep with the feeling of being suffocated, along with the dried blood around the patient's lips (likely due to coughing up blood-tinged sputum), are classic indicators of left-sided congestive heart failure (CHF). In left-sided CHF, stroke volume (the amount of blood ejected from the ventricle per contraction) is decreased secondary to a weakened or damaged myocardium. Decreased stroke volume causes blood to regurgitate into the upper chamber of the heart and ultimately backs up into the lungs and causes pulmonary edema. A 71-year-old male presents with chest pain and shortness of breath. He is conscious, but confused, and is profusely diaphoretic. He has weakly palpable radial pulses, a BP of 70/40 mm Hg, and diffuse crackles in all lung fields. You administer high-flow oxygen and apply the cardiac monitor, which reveals sinus tachycardia. The closest appropriate hospital is 40 miles away. Which of the following is the MOST appropriate next action? A: Give 20 mL/kg fluid boluses. B: Obtain a 12-lead ECG tracing. C: Perform a head-to-toe exam. D: Begin an infusion of dopamine - ANS*D: Begin an infusion of dopamine* Reason: The patient in this scenario has likely experienced an acute myocardial infarction and is now in cardiogenic shock (pump failure). Cardiogenic shock is characterized by general signs of shock (eg, tachycardia, diaphoresis), hypotension, altered mental status, and pulmonary congestion—a sign of significant left ventricular damage and decreased stroke volume. After ensuring airway patency and adequate oxygenation and ventilation, your priority is to improve perfusion. Crystalloid fluid boluses, at least not large fluid boluses (ie, 20 mL/kg), are not appropriate for this patient; they may exacerbate his pulmonary edema and further impair pulmonary respiration. Dopamine is a more appropriate intervention. In a dosing range of 5 to 10 µg/kg/min, dopamine possesses positive inotropic effects, which increases myocardial contractility and may improve cardiac output. Rapid transport for this patient is essential; because of your extended transport time, start the dopamine infusion en route. Unfortunately, true cardiogenic shock has a high mortality rate. A 71-year-old male presents with chest pain and shortness of breath. He is conscious, but confused, and is profusely diaphoretic. He has weakly palpable radial pulses, a BP of 70/40 mm Hg, and diffuse crackles in all lung fields. You administer high-flow oxygen and apply the cardiac monitor, which reveals sinus tachycardia. The closest appropriate hospital is 40 miles away. Which of the following is the MOST appropriate next action? - ANSBegin an infusion of dopamine. The patient in this scenario has likely experienced an acute myocardial infarction and is now in cardiogenic shock (pump failure). Cardiogenic shock is characterized by general signs of shock (eg, tachycardia, diaphoresis), hypotension, altered mental status, and pulmonary congestion—a sign of significant left ventricular damage and decreased stroke volume. After ensuring airway patency and adequate oxygenation and ventilation, your priority is to improve perfusion. Crystalloid fluid boluses, at least not large fluid boluses (ie, 20 mL/kg), are not appropriate for this patient; they may exacerbate his pulmonary edema and further impair pulmonary respiration. Dopamine is a more appropriate intervention. In a dosing range of 5 to 10 µg/kg/min, dopamine possesses positive inotropic effects, which increases myocardial contractility and may improve cardiac output. Rapid transport for this patient is essential; because of your extended transport time, start the dopamine infusion en route. Unfortunately, true cardiogenic shock has a high mortality rate. A 72-year-old male presents with an acute onset of confusion, slurred speech, and decreased movement of his right arm. The patient's wife tells you that this began about 20 minutes ago, and that he was fine before that. He has type II diabetes, hypertension, and atrial fibrillation. Given this patient's clinical presentation and past medical history, you should be MOST suspicious that he has: A: a space-occupying intracranial lesion. B: acute hypoglycemia. C: an acute epidural hemorrhage. D: an occluded cerebral artery - ANS*D: an occluded cerebral artery* Reason: Acute ischemic stroke, which is caused by an occluded cerebral artery, is characterized by an acute onset of confusion, slurred speech, facial droop, and unilateral weakness (hemiparesis), among other signs. This patient has two major risk factors for a stroke: hypertension and atrial fibrillation (A-Fib). Although hypertension could be a contributing factor, it is more likely that his A-Fib resulted in the stroke. In A-Fib, a small blood clot can dislodge from the wall of the fibrillating atria, enter the systemic circulation, and occlude a cerebral artery. An epidural hemorrhage is unlikely; it is generally the result of blunt head trauma—most often to the temporal lobe. Furthermore, patients with an epidural hemorrhage tend to deteriorate rapidly and exhibit signs of increased intracranial pressure. Epidural hemorrhage is most often the result of injury to the middle meningeal artery, which bleeds rapidly. Hypoglycemia can also present with acute confusion and slurred speech; however, hemiparesis is a less common finding. Clearly, you should assess the blood glucose level of any patient with an altered mental status. Patients with a space-occupying intracranial lesion (eg, brain tumor) typically have a slow onset and insidious progression of symptoms—often over a period of months. In some patients with a brain tumor, a seizure may be the only presenting clinical manifestation. A 72-year-old male presents with an acute onset of confusion, slurred speech, and decreased movement of his right arm. The patient's wife tells you that this began about 20 minutes ago, and that he was fine before that. He has type II diabetes, hypertension, and atrial fibrillation. Given this patient's clinical presentation and past medical history, you should be MOST suspicious that he has: A: an acute epidural hemorrhage. B: acute hypoglycemia. C: a space-occupying intracranial lesion. D: an occluded cerebral artery. - ANS*D: an occluded cerebral artery.* Reason: Acute ischemic stroke, which is caused by an occluded cerebral artery, is characterized by an acute onset of confusion, slurred speech, facial droop, and unilateral weakness (hemiparesis), among other signs. This patient has two major risk factors for a stroke: hypertension and atrial fibrillation (A-Fib). Although hypertension could be a contributing factor, it is more likely that his A-Fib resulted in the stroke. In A-Fib, a small blood clot A middle-aged man is found unresponsive, pulseless, and apneic. His cardiac arrest was not witnessed, although his skin is still warm to the touch. You should: - ANSBegin immediate high-quality CPR. The first and most crucial intervention for any patient in cardiac arrest is immediate high-quality CPR. With CPR ongoing, you or your partner can apply the defibrillation pads and assess the patient's cardiac rhythm. If a shock is indicated, deliver it and immediately resume CPR, starting with chest compressions. During the 2-minute cycles of CPR, vascular access can be obtained, cardiac drugs can be administered, and the patient's airway can be secured with an advanced device if necessary. It is absolutely critical to minimize interruptions in chest compressions; if you must interrupt compressions, do so for no longer than 10 seconds. The precordial thump is not indicated for unwitnessed cardiac arrest; it may be considered for patients with witnessed V-Tach, however, but has a low success rate. A middle-aged man presents with chest discomfort, shortness of breath, and nausea. You give him supplemental oxygen and continue your assessment. As your partner is attaching the ECG leads, you should: A: establish vascular access. B: administer 0.4 mg of nitroglycerin. C: administer up to 325 mg of aspirin. D: administer 2 to 4 mg of morphine IM. - ANS*C: administer up to 325 mg of aspirin* Reason: Since oxygen has already been administered to this patient and your partner is attaching the ECG leads, you should administer aspirin (160 to 325 mg, non-enteric- coated). Early administration of aspirin has clearly been shown to reduce mortality and morbidity in patients experiencing an acute coronary syndrome (ACS). After establishing vascular access, you should assess his vital signs and then administer 0.4 mg of nitroglycerin (up to 3 doses, 5 minutes apart), provided that his systolic BP is greater than 90 mm Hg. If 3 doses of nitroglycerin fail to completely relieve his chest discomfort, consider administering 2 to 4 mg of morphine IV, provided that his systolic BP remains above 90 mm Hg. A middle-aged man presents with chest discomfort, shortness of breath, and nausea. You give him supplemental oxygen and continue your assessment. As your partner is attaching the ECG leads, you should: - ANSAdminister up to 325 mg of aspirin. Since oxygen has already been administered to this patient and your partner is attaching the ECG leads, you should administer aspirin (160 to 325 mg, non-enteric- coated). Early administration of aspirin has clearly been shown to reduce mortality and morbidity in patients experiencing an acute coronary syndrome (ACS). After establishing vascular access, you should assess his vital signs and then administer 0.4 mg of nitroglycerin (up to 3 doses, 5 minutes apart), provided that his systolic BP is greater than 90 mm Hg. If 3 doses of nitroglycerin fail to completely relieve his chest discomfort, consider administering 2 to 4 mg of morphine IV, provided that his systolic BP remains above 90 mm Hg. A patient experiencing a right ventricular infarction would be expected to present with: A: hypertension and tachycardia. B: ST elevation in leads II, III, and aVF. C: severe pulmonary edema and hemoptysis. D: greater than 2-mm ST depression in lead V1. - ANS*B: ST elevation in leads II, III, and aVF.* Reason: A right ventricular infarction (RVI) should be suspected when a patient presents with ECG changes indicative of an inferior wall injury pattern (equal to or greater than 1-mm ST elevation in leads II, III, and aVF; reciprocol ST depression and T wave inversion in leads I and aVL) AND has equal to or greater than 1-mm ST elevation in lead V4R when a right-sided 12-lead ECG is obtained. Patients experiencing an RVI are preload dependent and often present with hypotension; therefore, vasodilators (eg, nitroglycerin, morphine) should be avoided. Instead, IV fluid boluses should be given to maintain adequate perfusion. Other signs of an RVI include jugular venous distention and peripheral edema. Pulmonary edema and coughing up blood (hemoptysis) are indicative of left ventricular failure. A patient experiencing a right ventricular infarction would be expected to present with: - ANSST elevation in leads II, III, and aVF. A right ventricular infarction (RVI) should be suspected when a patient presents with ECG changes indicative of an inferior wall injury pattern (equal to or greater than 1-mm ST elevation in leads II, III, and aVF; reciprocol ST depression and T wave inversion in leads I and aVL) AND has equal to or greater than 1-mm ST elevation in lead V4R when a right-sided 12-lead ECG is obtained. Patients experiencing an RVI are preload dependent and often present with hypotension; therefore, vasodilators (eg, nitroglycerin, morphine) should be avoided. Instead, IV fluid boluses should be given to maintain adequate perfusion. Other signs of an RVI include jugular venous distention and peripheral edema. Pulmonary edema and coughing up blood (hemoptysis) are indicative of left ventricular failure. A patient with acute chest discomfort displays the cardiac rhythm shown below. Which of the following is the MOST detrimental effect that this rhythm can have on the patient? - ANSIncreased myocardial oxygen demand. The rhythm shown is sinus tachycardia. Any increase in cardiac workload, such as an increase in heart rate, contractility, or blood pressure, will increase the amount of oxygen that the myocardium demands and consumes. In patients experiencing an acute coronary syndrome (ie, unstable angina [UA], acute myocardial infarction [AMI]), this could extend the area of ischemia or infarction. A patient's medication regimen includes fluoxetine, Toprol, Proscar, lansoprazole, and Klonopin. Which of these medications is used to treat cardiovascular disorders? A: fluoxetine B: Toprol C: lansoprazole D: Proscar - ANS*B: Toprol* Reason: Toprol (metaprolol) is a commonly prescribed beta-blocker used to treat various cardiovascular conditions, including hypertension and tachydysrhythmias. Proscar (finasteride) is used to treat benign prostatic hyperplasia (BPH). Fluoxetine (Prozac) is a selective serotonin reuptake inhibitor (SSRI) antidepressant. It is used to treat conditions such as depression, generalized anxiety disorder, and obsessive-compulsive disorder (OCD). Lansoprazole (Prevacid)—a proton pump inhibitor—is used to treat conditions such as heartburn, acid reflux disease, and ulcers. Clonazepam (Klonopin) is a benzodiazepine sedative-hypnotic; it is used to treat anxiety. A patient's medication regimen includes fluoxetine, Toprol, Proscar, lansoprazole, and Klonopin. Which of these medications is used to treat cardiovascular disorders? - ANSToprol. Toprol (metaprolol) is a commonly prescribed beta-blocker used to treat various cardiovascular conditions, including hypertension and tachydysrhythmias. Proscar (finasteride) is used to treat benign prostatic hyperplasia (BPH). Fluoxetine (Prozac) is a selective serotonin reuptake inhibitor (SSRI) antidepressant. It is used to treat conditions such as depression, generalized anxiety disorder, and obsessive-compulsive disorder (OCD). Lansoprazole (Prevacid)—a proton pump inhibitor—is used to treat conditions such as heartburn, acid reflux disease, and ulcers. Clonazepam (Klonopin) is a benzodiazepine sedative-hypnotic; it is used to treat anxiety. A PT with hyperkalemia will present with what abnormal ecg findings - ANST wave A PT with hypokalemia will present with what abnormal ecg findings - ANSU wave A PT with stroke like symptoms that resolve within 24 hours without per any damage is called - ANSTIA A transmural myocardial infarction is defined as: A: an MI that involves the entire thickness of the left ventricular wall from endocardium to epicardium. B: multiple areas of myocardial necrosis confined to the inner one third to one half of the left ventricular wall. C: an MI that occurs without gross ST segment elevation or the presence of a pathologic Q wave. arterial blood gas (ABG) values. Synchronized cardioversion is indicated for hemodynamically unstable patients with wide and narrow complex tachycardias, not PEA. After performing synchronized cardioversion on an unstable patient with a wide-complex tachycardia, you look at the monitor and see coarse ventricular fibrillation. The patient is unresponsive, apneic, and pulseless. You should: A: prepare to intubate the patient as your partner begins CPR. B: start CPR, ensure the synchronize mode is off, and defibrillate. C: begin CPR, establish vascular access, and give amiodarone. D: perform CPR for 2 minutes and then cardiovert with 100 joules. - ANS*B: start CPR, ensure the synchronize mode is off, and defibrillate* Reason: If a patient develops ventricular fibrillation (V-Fib) or pulseless ventricular tachycardia (V-Tach) following synchronized cardioversion, immediately begin CPR (even if it's just for a short period of time), ensure that the monitor/defibrillator is not in synchronize mode, and defibrillate as soon as possible. CPR should be ongoing as the defibrillator is charging in order to avoid unnecessary delays in performing chest compressions. The synchronize mode must be turned off prior to defibrillation or the device will not deliver a shock; this is because there are no R waves to synchronize with in V-Fib. Vascular access (IV or IO), advanced airway management, and pharmacologic therapy should be performed during the 2-minute cycles of CPR; they are not an immediate priority during early cardiac arrest. After performing synchronized cardioversion on an unstable patient with a wide-complex tachycardia, you look at the monitor and see coarse ventricular fibrillation. The patient is unresponsive, apneic, and pulseless. You should: - ANSStart CPR, ensure the synchronize mode is off, and defibrillate. If a patient develops ventricular fibrillation (V-Fib) or pulseless ventricular tachycardia (V-Tach) following synchronized cardioversion, immediately begin CPR (even if it's just for a short period of time), ensure that the monitor/defibrillator is not in synchronize mode, and defibrillate as soon as possible. CPR should be ongoing as the defibrillator is charging in order to avoid unnecessary delays in performing chest compressions. The synchronize mode must be turned off prior to defibrillation or the device will not deliver a shock; this is because there are no R waves to synchronize with in V-Fib. Vascular access (IV or IO), advanced airway management, and pharmacologic therapy should be performed during the 2-minute cycles of CPR; they are not an immediate priority during early cardiac arrest. afterload - ANSthe force of driving blood out of the heart against the high resistance of systemic arteries Amiodarone - ANS-Blocks sodium channels and myocardial potassium channels, delaying repolarization and increasing the duration of action potential An ECG shows a wide complex rhythm at a rate of 36 beats per minute, what part of the heart is this originating - ANSpurkinje fibers An elderly man is apneic and pulseless. The ECG shows the following rhythm, which you should interpret as: - ANSSinus tachycardia. The rhythm shown is sinus tachycardia at a rate of approximately 100 to 110 beats/min. First-degree AV block is characterized by a PR interval that is greater than 0.20 seconds, the normal being 0.12 to 0.20 seconds (120 to 200 milliseconds). The fact that the patient does not have a pulse indicates pulseless electrical activity (PEA). PEA is not a specific rhythm; it is a condition in which a pulseless, apneic patient presents with an organized cardiac rhythm (except for pulseless V-Tach). An older man is suddenly awakened in the middle of the night, gasping for air. He is extremely restless and pale, and is coughing up blood. His clinical presentation is MOST consistent with: A: left side heart failure. B: unstable angina. C: right side heart failure. D: gastrointestinal bleed. - ANS*A: left side heart failure.* Reason: Waking up in the middle of the night with severe difficulty breathing (paroxysmal nocturnal dyspnea [PND]) and coughing up blood or blood-tinged sputum (hemoptysis) are consistent with left-sided heart failure and pulmonary edema. Right-sided heart failure typically does not present with respiratory distress; it commonly manifests with jugular venous distention and peripheral edema. Shortness of breath and hemoptysis are not consistent with a gastrointestinal (GI) bleed; signs of a GI bleed include abdominal pain, vomiting up blood (hematemesis), which may be bright red or have a coffee-ground appearance; dark, tarry stools (melena); or bright red blood in the stool (hematochezia). Because left-sided heart failure can be caused by other factors, such as a long history of poorly-controlled hypertension, angina may or may not be present. An older man is suddenly awakened in the middle of the night, gasping for air. He is extremely restless and pale, and is coughing up blood. His clinical presentation is MOST consistent with: - ANSLeft side heart failure. Waking up in the middle of the night with severe difficulty breathing (paroxysmal nocturnal dyspnea [PND]) and coughing up blood or blood-tinged sputum (hemoptysis) are consistent with left-sided heart failure and pulmonary edema. Right-sided heart failure typically does not present with respiratory distress; it commonly manifests with jugular venous distention and peripheral edema. Shortness of breath and hemoptysis are not consistent with a gastrointestinal (GI) bleed; signs of a GI bleed include abdominal pain, vomiting up blood (hematemesis), which may be bright red or have a coffee-ground appearance; dark, tarry stools (melena); or bright red blood in the stool (hematochezia). Because left-sided heart failure can be caused by other factors, such as a long history of poorly-controlled hypertension, angina may or may not be present. An overdose on what medication could result in bradycardia and hypertension in the patient? - ANSNorepinephrine aorta - ANSthe largest artery in the body Aortic semilunar valve - ANSseparates the left ventricle from the pulmonary artery Appendicitis S/S - ANSRLQ abdominal pain or cramping, nausea, vomiting, chills, low grade fever Appropriate treatment for asystole includes: A: vasopressin every 3 to 5 minutes and tracheal intubation. B: supraglottic airway placement and antidysrhythmic therapy. C: transcutaneous cardiac pacing and epinephrine 1:10,000. D: epinephrine 1:10,000 and advanced airway management. - ANS*D: epinephrine 1:10,000 and advanced airway management.* Reason: Appropriate treatment for a patient in asystole includes high-quality CPR with minimal interruptions, vascular access, 1 mg of epinephrine 1:10,000 every 3 to 5 minutes, advanced airway management (eg, ET tube, multilumen airway, supraglottic airway), and assessing for and ruling out potentially reversible causes (Hs and Ts). Vasopressin may be given in a one-time dose of 40 units to replace the first or second dose of epinephrine, but not both. Transcutaneous cardiac pacing (TCP) has not shown to be beneficial for patients in asystole and is not recommended. Antidysrhythmic drugs, such as amiodarone and lidocaine, are indicated for patients with ventricular fibrillation or pulseless ventricular tachycardia; they are not given to patients with asystole. Appropriate treatment for asystole includes: - ANSEpinephrine 1:10,000 and advanced airway management. Appropriate treatment for a patient in asystole includes high-quality CPR with minimal interruptions, vascular access, 1 mg of epinephrine 1:10,000 every 3 to 5 minutes, advanced airway management (eg, ET tube, multilumen airway, supraglottic airway), and assessing for and ruling out potentially reversible causes (Hs and Ts). Vasopressin may be given in a one-time dose of 40 units to replace the first or second dose of epinephrine, but not both. Transcutaneous cardiac pacing (TCP) has not shown to be beneficial for patients in asystole and is not recommended. Antidysrhythmic drugs, such B: blocking sympathetic activity. C: opposing the vagus nerve. D: stimulating alpha receptors. - ANS*C: opposing the vagus nerve* Reason: Atropine sulfate is a parasympathetic blocker (parasympatholytic, vagolytic). It is used to increase the heart rate by opposing the vagus nerve when excessive parasympathetic (vagal) tone causes symptomatic bradycardia. Alpha adrenergic agonists, such as norepinephrine (Levophed), primarily stimulate alpha-1 receptors and cause vasoconstriction. Drugs such as propranolol (Inderal) and prazosin (Minipress) block sympathetic nervous system activity by binding to beta and alpha receptors, respectively. Beta receptor blockade causes a decrease in heart rate (negative chronotropy), a decrease in contractility (negative inotropy), and a decrease in electrical conduction velocity (negative dromotropy). Alpha receptor blockade causes vasodilation, and a subsequent decrease in blood pressure. Drugs that increase cardiac contractility, such as dopamine (Intropin), do so through their positive inotropic effects. Atropine sulfate exerts its therapeutic effect by: - ANSOpposing the vagus nerve. Atropine sulfate is a parasympathetic blocker (parasympatholytic, vagolytic). It is used to increase the heart rate by opposing the vagus nerve when excessive parasympathetic (vagal) tone causes symptomatic bradycardia. Alpha adrenergic agonists, such as norepinephrine (Levophed), primarily stimulate alpha-1 receptors and cause vasoconstriction. Drugs such as propranolol (Inderal) and prazosin (Minipress) block sympathetic nervous system activity by binding to beta and alpha receptors, respectively. Beta receptor blockade causes a decrease in heart rate (negative chronotropy), a decrease in contractility (negative inotropy), and a decrease in electrical conduction velocity (negative dromotropy). Alpha receptor blockade causes vasodilation, and a subsequent decrease in blood pressure. Drugs that increase cardiac contractility, such as dopamine (Intropin), do so through their positive inotropic effects. automaticity - ANSthe ability of heart muscle to generate its own electrical impulses without nerve stimulation Battery ` - ANSIs the act of causing harm to someone Becks triad is associated with ___ - ANScardiac tamponade Becks Triad signs - ANSJVD, narrowing pulse pressure, hypotension and muffled heart tones benign early repolarization - ANScharacterized by ST-elevation or fishhook appearance at the J point and concave ST segment often seen exclusively in left precordial leads; there are never any reciprocal changes beta blockers - ANSdecrease the rate and strength of cardiac contractions thereby decreasing the hearts demand for oxygen Beta Blockers (lol's) - ANS-Bind to beta-adrenoceptors and thereby block the binding of of norepinephrine and epinephrine to these receptors -Inhibits normal sympathetic effects that act through these receptors (sympatholytic) blood pressure - ANSthe pressure exerted by the blood against the arterial walls generated by repeated forceful contractions of the left ventricle which keeps blood flowing through the body. Magnitude is influenced by caridac output, blood volume and relative constriction/dilation of the arteries Brugada syndrome - ANSgenetic disorder involving the sodium channels in the heart; often unaware of condition until sudden onset. Incomplete RBBB and ST-segment elevation that aggressively returns to baseline bundle branch block - ANSQRS complex with a bizarre appearance and a long duration calcium channel blockers - ANSblock the influx of calcium ions into cadiac muscle and relieve angina by preventing coronary artery spasms and weakening cardiac contraction thereby decreasing myocardial oxygen demand. Calcium Channel Blockers (pine's) - ANS-Block calcium entry into cardiac cells causing vascular smooth muscle relaxation, decreased myocardial force generation, decreased heart rate, and decreased conduction velocity particularly at the AV node Calcium Chloride - ANS-Increases cardiac contractile state (positive inotropic effect) -May enhance ventricular automaticity Calcium Gluconate - ANS-Counteracts the toxicity of hyperkalemia by stabilizing the membranes of the cardiac cells, reducing the likelihood of fibrillation cardiac output - ANSamount of blood that is pumped out by either ventircles. Normal is ~ 5-6L/min. CO= SV X HR cardiac tamponade - ANSwhen excessive fluid accumulates within the pericardium, limiting the hearts ability to expand fully after each contraction and resulting in reduced CO. Diagnosed with gradual dyspnea and weakness or traumatic chest pain. There will be an initial systolic drop then the diastolic will slowly rise (narrowing pulse pressure) followed by an increase in heart rate and muffled heart sounds. May have JVD. cardiogenic shock - ANSwhen the heart is so damaged that it can no longer pump a volume of blood sufficient for tissue perfusion Cardioversion involves delivering a shock that is synchronized to occur during the: A: P wave. B: downslope of the T wave. C: upslope of the T wave. D: R wave. - ANS*D: R wave.* Reason: Cardioversion involves delivering a shock that is synchronized to occur during the R wave, which is when the heart is absolutely refractory. This prevents the shock from occurring during the relative refractory period (the downslope of the T wave). Depolarization that occurs during the relative refractory period may induce a non- perfusing ventricular dysrhythmia, such as pulseless V-Tach or V-Fib. Synchronized cardioversion is indicated for patients with supraventricular or ventricular tachycardia who have a pulse, but are hemodynamically unstable. Cardioversion involves delivering a shock that is synchronized to occur during the: - ANSR wave. Cardioversion involves delivering a shock that is synchronized to occur during the R wave, which is when the heart is absolutely refractory. This prevents the shock from occurring during the relative refractory period (the downslope of the T wave). Depolarization that occurs during the relative refractory period may induce a non- perfusing ventricular dysrhythmia, such as pulseless V-Tach or V-Fib. Synchronized cardioversion is indicated for patients with supraventricular or ventricular tachycardia who have a pulse, but are hemodynamically unstable. cholecystitis is more common in which type of patient? - ANSIt is more common in women age 30-50 chronotropic effect - ANSalters the heart's rate of contraction claudication - ANSa severe pain in the calf muscle caused by a narrowing of the arteries in this muscle and leading to a painfl limp collateral circulation - ANSdevelops in response to the early stages of coronary heart disease during which the inside diameter of the coronary arteries begins to narrow as plaque deposits on the walls. This is the formation of additional blood vessels connecting arterioles originating in other blood vessels in order to increase oxygenated blood delivery to the myocardium. conductivity - ANSallows cardiac cells to pass an electrical impulse from one cell to another congestive heart failure - ANSwhen the heart is unable to pump powerfully or fast enough to empty its chambers which results in blood backing up into the systemic circuit, the pulmonary circuit, or both The 2010 guidelines for CPR and emergency cardiac care (ECC) have added a fifth link to the chain of survival, integrated post-arrest care. In addition to supporting the patient's airway and ventilatory status and supporting his or her blood pressure with IV fluid boluses or an inotropic agent (ie, dopamine), you should assess for and correct any glucose abnormalities. If the patient is unable to follow verbal commands or remains comatose following return of spontaneous circulation (ROSC), therapeutic hypothermia (89.6°F to 93.2°F [32°C to 34°C]) has been shown to improve neurologic recovery and should be considered (follow your local protocols regarding therapeutic hypothermia). Once ROSC has been established, you should continue to ventilate the adult patient at a rate of 10 to 12 breaths/min (one breath every 5 to 6 seconds) if he or she remains apneic. DO NOT hyperventilate the patient as this may impair venous return to the heart and compromise cardiac output. If the patient is able to follow verbal commands following ROSC, obtain a 12-lead ECG tracing and assess for signs of acute MI (ie, ST elevation). Depending on your transport time, you may consider starting a maintenance infusion of the antidysrhythmic drug that was administered during the arrest, which in this case, would be amiodarone (1 mg/min). During ventricular systole what is the blood pumped through? - ANSPulmonary and aortic valve During your SAMPLE history of an elderly man, he tells you that his cardiologist told him that he has an "irregular heartbeat." His medications include warfarin sodium and digoxin. On the basis of this information, what underlying cardiac rhythm should you suspect? A: Sinus dysrhythmia B: Atrial fibrillation C: AV heart block D: Atrial tachycardia - ANS*B: Atrial fibrillation* Reason: Patients with atrial fibrillation (A-Fib) are commonly prescribed digoxin (a digitalis preparation) and warfarin sodium (Coumadin), which is a blood thinner. As the atria fibrillate, blood has a tendency to stagnate and form microemboli that can be ejected from the heart and occlude a pulmonary, cerebral, or coronary artery. During your SAMPLE history of an elderly man, he tells you that his cardiologist told him that he has an "irregular heartbeat." His medications include warfarin sodium and digoxin. On the basis of this information, what underlying cardiac rhythm should you suspect? - ANSAtrial fibrillation. Patients with atrial fibrillation (A-Fib) are commonly prescribed digoxin (a digitalis preparation) and warfarin sodium (Coumadin), which is a blood thinner. As the atria fibrillate, blood has a tendency to stagnate and form microemboli that can be ejected from the heart and occlude a pulmonary, cerebral, or coronary artery. dysconjugate gaze - ANSDeviation of the eyes to the opposite sides at rest implies damage to the brainstem Eccymosis - ANSBruising that looks identical to a contusion, but is not the result of internal bleeding. ECG indicators of Wolff-Parkinson-White (WPW) syndrome include: A: tall P waves, QT interval prolongation, and tachycardia. B: narrow QRS complexes and peaked T waves. C: short PR intervals, delta waves, and QRS widening. D: delta waves, flattened T waves, and bradycardia. - ANS*C: short PR intervals, delta waves, and QRS widening* Reason: Wolff-Parkinson-White (WPW) syndrome is a condition in which accessory pathways— called the bundle of Kent—bypass the atrioventricular (AV) node, causing the ventricles to depolarize earlier than normal (preexcitation). Because the normal delay at the AV node does not occur, the PR intervals in patients with WPW are usually less than 0.12 seconds (120 ms). When conduction occurs down the AV node and simultaneously along the bundle of Kent in an anterograde fashion, the two waves of depolarization meet (fusion). This manifests on the ECG as a delta wave—slurring or notching at the beginning of the QRS complex—which may cause QRS widening. The bundle of Kent is a potential site for a reentry circuit because it allows continued transmission of an electrical impulse from the atria to the ventricles. Therefore, patients with WPW are prone to reentry tachycardias—most notably, AV reentry supraventricular tachycardia (SVT). ECG indicators of Wolff-Parkinson-White (WPW) syndrome include: A: tall P waves, QT interval prolongation, and tachycardia. B: narrow QRS complexes and peaked T waves. C: delta waves, flattened T waves, and bradycardia. D: short PR intervals, delta waves, and QRS widening. - ANS*D: short PR intervals, delta waves, and QRS widening* Reason: Wolff-Parkinson-White (WPW) syndrome is a condition in which accessory pathways— called the bundle of Kent—bypass the atrioventricular (AV) node, causing the ventricles to depolarize earlier than normal (preexcitation). Because the normal delay at the AV node does not occur, the PR intervals in patients with WPW are usually less than 0.12 seconds (120 ms). When conduction occurs down the AV node and simultaneously along the bundle of Kent in an anterograde fashion, the two waves of depolarization meet (fusion). This manifests on the ECG as a delta wave—slurring or notching at the beginning of the QRS complex—which may cause QRS widening. The bundle of Kent is a potential site for a reentry circuit because it allows continued transmission of an electrical impulse from the atria to the ventricles. Therefore, patients with WPW are prone to reentry tachycardias—most notably, AV reentry supraventricular tachycardia (SVT). ECG indicators of Wolff-Parkinson-White (WPW) syndrome include: - ANSShort PR intervals, delta waves, and QRS widening. Wolff-Parkinson-White (WPW) syndrome is a condition in which accessory pathways— called the bundle of Kent—bypass the atrioventricular (AV) node, causing the ventricles to depolarize earlier than normal (preexcitation). Because the normal delay at the AV node does not occur, the PR intervals in patients with WPW are usually less than 0.12 seconds (120 ms). When conduction occurs down the AV node and simultaneously along the bundle of Kent in an anterograde fashion, the two waves of depolarization meet (fusion). This manifests on the ECG as a delta wave—slurring or notching at the beginning of the QRS complex—which may cause QRS widening. The bundle of Kent is a potential site for a reentry circuit because it allows continued transmission of an electrical impulse from the atria to the ventricles. Therefore, patients with WPW are prone to reentry tachycardias—most notably, AV reentry supraventricular tachycardia (SVT). ejection click - ANShigh-pitched sound indicating a dilated pulmonary artery or septal defect ejection fraction - ANSpercentage of blood that leaves the heart each time that it contracts. Normal range is 55-70% but may be lower with damage endocarditis - ANSinfection of the inner lining of the heart characterized by inflammation of the inside lining of the heart chambers including the valves; almost exclusively caused by the straph or strep virus endocardium - ANSthin membrane that lines the inside of the hearts cavities and forms the valves; vital to reduction of turbulance. epicardium - ANSthin membrane that forms the outermost layer of the heart Epinephrine - ANS-Direct-acting alpha and beta agonist -Blocks histamine receptors Excessive menstrual flow is known as what? - ANSMenorrhagia excitability - ANSallows cardiac cells to respond to an electrical impulse Fentanyl Citrate - ANS-Binds to opiate receptors, producing analgesia and euphoria D: ST segment depression in leads II, III, and aVF. - ANSC: ST segment elevation in leads V1 through V4. Reason: ST segment elevation that is equal to or greater than 1-mm in two or more contiguous leads indicates myocardial injury (eg, an acute MI in progress). ST segment depression and/or dynamic T wave inversion indicates myocardial ischemia. Leads V1 and V2 view the interventricular septum; leads V3 and V4 view the anterior wall of the left ventricle; leads V5, V6, I, and aVL view the lateral wall of the left ventricle; and leads II, III, and aVF view the inferior wall of the left ventricle. Therefore, if a patient is experiencing acute injury involving the interventricular septum and anterior wall of the left ventricle (anteroseptal injury), you would expect the 12-lead ECG tracing to reveal ST segment elevation is leads V1 through V4. It is important to note, however, that an absence of ST elevation does not definitively rule out acute myocardial infarction. If a patient was experiencing acute injury involving the interventricular septum and anterior wall of the left ventricle, you would expect the 12-lead ECG to reveal: - ANSST segment elevation in leads V1 through V4. ST segment elevation that is equal to or greater than 1-mm in two or more contiguous leads indicates myocardial injury (eg, an acute MI in progress). ST segment depression and/or dynamic T wave inversion indicates myocardial ischemia. Leads V1 and V2 view the interventricular septum; leads V3 and V4 view the anterior wall of the left ventricle; leads V5, V6, I, and aVL view the lateral wall of the left ventricle; and leads II, III, and aVF view the inferior wall of the left ventricle. Therefore, if a patient is experiencing acute injury involving the interventricular septum and anterior wall of the left ventricle (anteroseptal injury), you would expect the 12-lead ECG tracing to reveal ST segment elevation is leads V1 through V4. It is important to note, however, that an absence of ST elevation does not definitively rule out acute myocardial infarction. If the pupils are fixed and dilated there has been an injury to which part of the CNS - ANSThe brainstem Immediately after establishing a return of spontaneous circulation in a woman with ventricular fibrillation of short duration, you should: A: obtain a 12-lead ECG to assess for cardiac damage. B: assess her ventilatory status and treat accordingly. C: assess her blood pressure and treat if needed. D: establish vascular access and give amiodarone. - ANS*B: assess her ventilatory status and treat accordingly.* Reason: Your first action after establishing return of spontaneous circulation (ROSC) in a patient —regardless of his or her arrest rhythm and duration—is to assess the patient's ventilatory status. If the patient is not breathing or is breathing inadequately, provide ventilatory support. After assessing and managing airway and breathing, assess the patient's blood pressure and stabilize it if it is low. Airway and circulatory support are critical following ROSC; inadequate ventilation and/or hypotension following cardiac arrest may lead to a recurrence of cardiac arrest. Depending on your local protocols, IV amiodarone may be given following ROSC. After assessing and maintaining respiratory and circulatory functions, obtain a 12-lead ECG if time allows. If the patient remains comatose following ROSC, consider inducing therapeutic hypothermia. Immediately after establishing a return of spontaneous circulation in a woman with ventricular fibrillation of short duration, you should: - ANSAssess her ventilatory status and treat accordingly. Your first action after establishing return of spontaneous circulation (ROSC) in a patient —regardless of his or her arrest rhythm and duration—is to assess the patient's ventilatory status. If the patient is not breathing or is breathing inadequately, provide ventilatory support. After assessing and managing airway and breathing, assess the patient's blood pressure and stabilize it if it is low. Airway and circulatory support are critical following ROSC; inadequate ventilation and/or hypotension following cardiac arrest may lead to a recurrence of cardiac arrest. Depending on your local protocols, IV amiodarone may be given following ROSC. After assessing and maintaining respiratory and circulatory functions, obtain a 12-lead ECG if time allows. If the patient remains comatose following ROSC, consider inducing therapeutic hypothermia. Immediately following return of spontaneous circulation, the paramedic should: A: reassess the patient's ventilatory status. B: induce therapeutic hypothermia. C: provide a bolus of normal saline solution. D: assess the patient's blood pressure - ANS*A: reassess the patient's ventilatory status* Reason: Immediately following return of spontaneous circulation (ROSC), as evidenced by the presence of a pulse, the paramedic should reassess the patient's ventilatory status and continue to treat accordingly. Remember, if an advanced airway is placed during cardiac arrest, ventilations are given at a rate of one breath every 6 to 8 seconds (8 to 10 breaths/min) with continuous chest compressions. However, if ROSC occurs and the patient remains apneic, you should deliver one breath every 5 to 6 seconds (10 to 12 breaths/min) for the adult, or one breath every 3 to 5 seconds (12 to 20 breaths/min) for infants and children. Next, assess the patient's BP and use crystalloid fluid boluses or an inotropic drug (eg, dopamine) to treat hypotension and maintain adequate perfusion. If the patient remains comatose following ROSC, therapeutic hypothermia should be considered. Follow your local protocols. Immediately following return of spontaneous circulation, the paramedic should: - ANSReassess the patient's ventilatory status. Immediately following return of spontaneous circulation (ROSC), as evidenced by the presence of a pulse, the paramedic should reassess the patient's ventilatory status and continue to treat accordingly. Remember, if an advanced airway is placed during cardiac arrest, ventilations are given at a rate of one breath every 6 to 8 seconds (8 to 10 breaths/min) with continuous chest compressions. However, if ROSC occurs and the patient remains apneic, you should deliver one breath every 5 to 6 seconds (10 to 12 breaths/min) for the adult, or one breath every 3 to 5 seconds (12 to 20 breaths/min) for infants and children. Next, assess the patient's BP and use crystalloid fluid boluses or an inotropic drug (eg, dopamine) to treat hypotension and maintain adequate perfusion. If the patient remains comatose following ROSC, therapeutic hypothermia should be considered. Follow your local protocols. In a patient with a tension pneumothorax the trachea will deviate toward which side? - ANSOpposite In addition to CPR, the recommended treatment sequence for an unresponsive, apneic, and pulseless patient with a regular, wide-complex cardiac rhythm at a rate of 40 beats/min includes: A: transcutaneous cardiac pacing and 1 mg of epinephrine every 3 to 5 minutes. B: 1 mg of epinephrine every 3 to 5 minutes and 1 gram of calcium chloride. C: 40 units of vasopressin every 10 minutes and treating reversible causes. D: 1 mg of epinephrine every 3 to 5 minutes and treating reversible causes. - ANS*D: 1 mg of epinephrine every 3 to 5 minutes and treating reversible causes* Reason: Pulseless electrical activity (PEA) exists when an unresponsive, apneic, pulseless patient presents with a regular cardiac rhythm. Treatment for PEA includes immediate high-quality CPR with minimal interruptions, obtaining vascular access (IV or IO), 1 mg of epinephrine every 3 to 5 minutes, advanced airway management (ie, ET tube, multilumen or supraglottic airway), and assessing for and treating reversible causes (Hs and Ts). Vasopressin, in a one-time dose of 40 units, can be given to replace the first or second dose of epinephrine, but not both. There are insufficient data to recommend transcutaneous pacing (TCP) for patients with bradycardic PEA or asystole, and the routine use of calcium chloride during cardiac arrest is not recommended. In addition to CPR, the recommended treatment sequence for an unresponsive, apneic, and pulseless patient with a regular, wide-complex cardiac rhythm at a rate of 40 beats/min includes: - ANS1 mg of epinephrine every 3 to 5 minutes and treating reversible causes. Pulseless electrical activity (PEA) exists when an unresponsive, apneic, pulseless patient presents with a regular cardiac rhythm. Treatment for PEA includes immediate high-quality CPR with minimal interruptions, obtaining vascular access (IV or IO), 1 mg of epinephrine every 3 to 5 minutes, advanced airway management (ie, ET tube, multilumen or supraglottic airway), and assessing for and treating reversible causes (Hs Dyspnea that occurs in the context of an acute coronary syndrome (ACS)—that is, unstable angina or acute myocardial infarction—should be assumed to be the result of left side congestive heart failure with resultant pulmonary congestion/edema. The majority of myocardial infarctions involve the left ventricle. The damage may be so extensive that myocardial contractility is impaired and blood backs up into the lungs. Cor pulmonale—acute right heart failure secondary to pulmonary hypertension— typically presents with systemic venous congestion (ie, JVD, peripheral edema), not pulmonary congestion. Anxiety is very common with ACS, and can potentially exacerbate the patient's condition due to increases in myocardial oxygen consumption and demand. In the interest of the patient, however, assume that any complaint of dyspnea in conjunction with ACS is the result of the worst case scenario—pulmonary edema and impaired oxygenation. In which of the following situations is transcutaneous cardiac pacing (TCP) clearly indicated? A: First-degree AV block in a patient with abdominal pain. B: Pulseless electrical activity at a rate of 50 beats/min. C: Third-degree AV block in a patient with pulmonary edema. D: Asystole, but only after 10 minutes of adequate CPR. - ANS*C: Third-degree AV block in a patient with pulmonary edema* Reason: Because third-degree AV block (complete heart block)—an inherently slow cardiac dysrhythmia—represents total atrioventricular dissociation, it is associated with hemodynamic compromise in most cases and should be treated with immediate transcutaneous cardiac pacing (TCP). Signs of hemodynamic compromise include ongoing chest pain, pulmonary edema, decreased level of consciousness, shortness of breath, and hypotension. First-degree AV block is typically a benign rhythm and is not commonly associated with hemodynamic compromise. Evidence has shown TCP to be of little or no benefit to patients with PEA or asystole, and it is clearly of no benefit to patients with prolonged asystole. In which of the following situations is transcutaneous cardiac pacing (TCP) clearly indicated? A: Pulseless electrical activity at a rate of 50 beats/min. B: Third-degree AV block in a patient with pulmonary edema. C: First-degree AV block in a patient with abdominal pain. D: Asystole, but only after 10 minutes of adequate CPR. - ANS*B: Third-degree AV block in a patient with pulmonary edema.* Reason: Because third-degree AV block (complete heart block)—an inherently slow cardiac dysrhythmia—represents total atrioventricular dissociation, it is associated with hemodynamic compromise in most cases and should be treated with immediate transcutaneous cardiac pacing (TCP). Signs of hemodynamic compromise include ongoing chest pain, pulmonary edema, decreased level of consciousness, shortness of breath, and hypotension. First-degree AV block is typically a benign rhythm and is not commonly associated with hemodynamic compromise. Evidence has shown TCP to be of little or no benefit to patients with PEA or asystole, and it is clearly of no benefit to patients with prolonged asystole. In which of the following situations is transcutaneous cardiac pacing (TCP) clearly indicated? - ANSThird-degree AV block in a patient with pulmonary edema. Because third-degree AV block (complete heart block)—an inherently slow cardiac dysrhythmia—represents total atrioventricular dissociation, it is associated with hemodynamic compromise in most cases and should be treated with immediate transcutaneous cardiac pacing (TCP). Signs of hemodynamic compromise include ongoing chest pain, pulmonary edema, decreased level of consciousness, shortness of breath, and hypotension. First-degree AV block is typically a benign rhythm and is not commonly associated with hemodynamic compromise. Evidence has shown TCP to be of little or no benefit to patients with PEA or asystole, and it is clearly of no benefit to patients with prolonged asystole. inferior vena cava - ANScollects deoxygenated blood from the lower portion of the body inotropic effect - ANSalters the contractility of the heart muscle intraventricular septum - ANSthin wall that separates the left and right sides of the heart J point - ANSthe junction point at the beginning of the ST segment Junctional tachycardia rate - ANS100+ Lefort I fracture - ANSFracture involving the maxillae Lefort II - ANSFractures involving the maxillae and nose Lefort III - ANSFractures of the maxillae, nose and orbits Andy will typically be a fracture off of the skull also left atrial enlargement - ANSlong P wave with a notched appearance and predominantly negative in lead 1 Left ventricular heart failure will cause a back flow of blood into which system? - ANSPulmonary circulation Lidocaine Hydrochloride - ANS-Decreases automaticity by slowing the rate of spontaneous phase 4 depolarization limb leads - ANSleads 1,2, 3, avR, aVL, and aVF Long QT syndrome - ANScondition characterized by a QT interval exceeding 450ms; indicates the heart is experiencing extra long refractory period making the ventricle more vulnerable to dysrhythmias low-pressure pump - ANSright side of the heart because it pumps against relatively low resistance of pulmonary circulation lumen - ANSthe opening within a blood vessel Lupus primarily effects what type of patient? - ANSFemales between the ages of 15-45 Magnesium Sulfate - ANS-Reduces striated muscle contractions and blocks peripheral neuromuscular transmission by reducing acetylcholine release at the myoneural junction -Can cause bronchodilation after beta-agonists and anti-cholinergics have been administered Mallory-Weiss syndrome - ANStear in the distal esophagus from retching in alcoholism or bulimia MAP calculation - ANSDBP + 1/3 (SBP - DBP) McBurney's point - ANSPain in RLQ with appendicitis melena - ANSblack tarry stool which indicates and upper GI bleed Mitral valve (bicuspid) - ANSseparates the left atrium from the left ventricle Modifiable risk factors for heart disease - ANShypertension, elevated cholesterol level, smoking, poor diet, obesity, sedentary lifestyle, oral contraceptives, hormone replacement therapy, and high stress. Morphine Sulfate - ANS-Alleviates pain through CNS action -Increases peripheral venous capacitance and decreases venous return -Decreases preload, afterload and myocadial oxygen demand murmur - ANSambiguous sound indicating turbulent blood flow through the valves Murphy's sign - ANSPain with palplation of gall bladder (seen with cholecystitis) myocarditis - ANSinflammation of the myocardium caused by viral, bacterial and fungal infections (can also be caused by trauma) P-wave - ANSthe depolarization of the atria and the brief pause as conduction is momentarily slowed through thr AV junction pacemaker - ANSthe area of conduction tissue in which the electrical activity arises at any given time that sets the pace for cardiac contraction Pain and bruising in the flank area - ANSGrey turners sign Palpitations - ANSthe sensation of an abnormally fast or irregular heart beat parasympathetic (cholinergic) nervous system - ANSrest and digest nervous system; uses primarily the vagus nerve. When activated things slow down, when blocked things cant. Uses the release of acetycholine to slow the heartl blocked by atropine. path of deoxygenated blood in the heart - ANSenters the right atrium and is pumped into the right ventricle which pumps it into the pulmonary artery for distribution into the lungs Pathology of altitude illness - ANSIt is attributed directly to exposure to reduced atmospheric pressure Patients who are non responsive to epinephrine during an anaphylactic reaction due to beta locker usage may be given what medication? - ANSGlucagon Pediatric adenosine dose - ANS0.1 mg/kg MR 0.2mg/kg Pediatric amiodarone cardiac arrest dose - ANS5 mg/kg Pediatric atropine dose - ANS0.02mg/kg Pediatric diephenhydramine dose - ANS pericardial friction rub - ANSto-and-fro sound indicating pericarditis pericardial knock - ANShigh-pitched sound during the diastole phase indicating a thickened pericardium limiting ventricular expansion pericarditis - ANSinflammation of the pericardial sac as the result of an infection or trauma. Positional chest pain (alleviated by sitting forward), shortness of breath, and history of recent infection or fever. Diffuse ST-segment elevation and a depressed PR segment, and concave ST segment (never any reciprocal changes) pericardium - ANSthe tough, fibrous sac surrounding the heart in order to protect it and provide lubrication between the heart and surrounding structures. phlebitis - ANSswelling and pain along the veins that can lead to the formation of blood clots called DVTs posterior ECG - ANSevaluates the electrical activity of the posterior wall of the left ventricle PR- interval - ANSrepresents the time required for an impulse to traverse the atria and AV junction and the amount of time the AV node delays transmission of atrial activity to the ventricles precordial leads - ANSleads V1-V6 Preexisting conditions for HHNK - ANSOften occurs in older patients with type 2 diabetes Prehospital treatment for pneumonia - ANSairway support, oxygen administration, ventilatory assistance as needed, IV fluids to support B/P, and to thin and loosen mucus, cardiac monitoring and sometimes bronchodilator drugs preload - ANSthe initial stretching of the cardiac myocytes prior to contraction of the left side of the heart Prinzmetal angina - ANSchest pain at rest caused by coronary artery vasospasm; more common in women in their 50s. PT with myasthenia gravis can not receive what drug? - ANSFentanyl pulmonary circulation - ANSconsists of all blood vessels between the right ventricle and left atrium pulmonary embolism - ANSS1Q3T3 pattern, new RBBB and ST- segment depression V1-V3. Pattern means deep S wave in lead 1, deep narrow Q wave in lead 3, and T- wave inversion lead 3. Pulmonary embolisms will typically present with this finding on an ECG - ANSS1Q3T3 pulmonary semilunar valve (pulmonic) - ANSseparates the right ventricle from the pulmonary artery; preventing backflow from the artery into the ventricle. pulmonary veins - ANScollect oxygen-rich blood and return it to the left atrium pulse deficit - ANSwhen a palpated radial pulse is less than an apical pulse pulsus alterans - ANSthe pulse alternates between strong and weak beats; typically indicates left ventricular systolic damage pulsus paradoxus - ANSan excessive drop in the systolic blood pressure with each inspired breath Purkinje fibers - ANSthousands of fibrils distributed through the ventricular muscle; during electrical impulse conduction here the ventricles contract simultaneously. 20-40 beats per minute QRS complex - ANSrepresents the depolarization of the ventricles relaxation phase - ANS0.52 seconds; left atrium fills passively with blood under venous pressure and about 80% of ventricular filling occurs as flood flows through the open tricuspid and mitral valves. Rhabdomyolysis - ANSWhen the body will break down muscle tissue and release the byproduct into the bloodstream rheumatic fever - ANSinflammatory disease caused by streptococcal bacteria that can cause a stenosis of the mitral or aortic valve, leading to heart complications right atrial enlargement - ANScharacterized by a colossal P wave in lead 2 Right ventricular heart failure will cause a back flow of blood into which system? - ANSSystemic circulation right-sided ECG - ANSevaluates the electrical activity of the right ventricle (generally lead V4R is the most indicative of right ventricular AMI) role of calcium in cardiac function - ANSrole in depolarization of pacemaker cells and myocardial contractility role of magnesium in cardiac function - ANSstabilizes the cell membrane; acts with potassium and opposes the actions of calcium role of potassium in cardiac function - ANSflows out of the cell to initiate repolarization Role of sodium in cardiac function - ANSflows into cell to initiate depolarization S1 - ANSLub-Tricuspid and Mitral valve close S2 - ANSDub-Aortic and pulmonic valves close S3 - ANSDah- ventricles filling(abnormal and may indicate a failing heart) S4 - ANSAtrial contraction ST segment - ANSthe end of ventricular depolarization and the beginning of ventricular repolarization stage 2 hemorrhage - ANS15-30% blood loss Stroke volume - ANSamount of blood pumped out by either ventricle in a single contraction. Normally 60-100mL but a healthy heart may do significantly more Sudden cardiac arrest in the adult population is MOST often secondary to: A: massive hypovolemia. B: respiratory failure. C: accidental electrocution. D: a cardiac dysrhythmia. - ANS*D: a cardiac dysrhythmia* Reason: Most cases of sudden cardiac arrest (SCA) in the adult population are secondary to a cardiac dysrhythmia, usually ventricular fibrillation (V-Fib). This fact underscores the criticality of early defibrillation. Respiratory failure is the most common cause of cardiac arrest in the pediatric population. Sudden cardiac arrest in the adult population is MOST often secondary to: - ANSA cardiac dysrhythmia. Most cases of sudden cardiac arrest (SCA) in the adult population are secondary to a cardiac dysrhythmia, usually ventricular fibrillation (V-Fib). This fact underscores the criticality of early defibrillation. Respiratory failure is the most common cause of cardiac arrest in the pediatric population. Sudden cardiac arrest in the adult population is MOST often secondary to:Sudden cardiac arrest in the adult population is MOST often secondary to: A: respiratory failure. B: a cardiac dysrhythmia. C: massive hypovolemia. D: accidental electrocution. - ANS*B: a cardiac dysrhythmia.* Reason: Most cases of sudden cardiac arrest (SCA) in the adult population are secondary to a cardiac dysrhythmia, usually ventricular fibrillation (V-Fib). This fact underscores the criticality of early defibrillation. Respiratory failure is the most common cause of cardiac arrest in the pediatric population. superior vena cava - ANScollects deoxygenated blood from the upper half of the body sympathetic (adrenergic) nervous system - ANSfight or flight nervous system; increases hr, cardiac muscle contractions and provides other responses to ensure that tissues increased oxygen demands are satisfied with increased CO. Commands are conveyed via norepi Symptoms of Lupus - ANSbutterfly shaped rash on chin & cheeks, pericarditis, pleuritis, extreme fatigue, painful or swollen joints, and kidney problems. systemic circulation - ANSconsists of all blood vessels between the left ventricle and left atrium systole - ANSthe period of time when the atria or ventricles are contracting Technique for performing a needle thoracostomy (or needle thoracentesis) - ANSThe needle can be inserted anteriorly in the second intercostal space in the midclavicular line just above the rib to avoid the nerve, artery, and vein The 3 components to the pediatric assessments - ANSAppearance, work of breathing and circulation The appropriate second dose and method of administration of amiodarone for a patient with refractory ventricular fibrillation is: A: 150 mg given over 10 minutes. B: 150 mg via rapid IV/IO push. C: 300 mg via rapid IV/IO push. D: 300 mg given over 10 minutes. - ANS*B: 150 mg via rapid IV/IO push.* Reason: The initial dose of amiodarone for a patient with refractory ventricular fibrillation or pulseless ventricular tachycardia is 300 mg via rapid IV or IO push. A second dose of 150 mg via rapid IV or IO push may be repeated one time in 5 minutes. For supraventricular tachycardia or ventricular tachycardia with a pulse, amiodarone should be given in a dose of 150 mg over 10 minutes; this same dose may be repeated as needed. The appropriate second dose and method of administration of amiodarone for a patient with refractory ventricular fibrillation is: - ANS150 mg via rapid IV/IO push. The initial dose of amiodarone for a patient with refractory ventricular fibrillation or pulseless ventricular tachycardia is 300 mg via rapid IV or IO push. A second dose of 150 mg via rapid IV or IO push may be repeated one time in 5 minutes. For supraventricular tachycardia or ventricular tachycardia with a pulse, amiodarone should be given in a dose of 150 mg over 10 minutes; this same dose may be repeated as needed. The AV node typically beats - ANS40-60bpm The cardiac cell required for cardiac muscle contraction which favors slow channels - ANSCalcium The classic triad of meningitis includes - ANSfever, altered mental status, and nucahl rigidity. The dosage of cardizem - ANS0.25mg/kg to a max of 10 mg slow IV push The dose of an epi drip - ANS2-10mcg/min The first drug of choice in an organophosphate poisoning - ANSAtropine should always be used first The first intervention in stable SVT - ANSVagal maneuvers The initial dose for pediatric cardioversion is - ANS0.5-1J/kg The initial dose of diltiazem for a 165-pound patient is approximately: A: 19 mg. B: 17 mg. C: 25 mg. D: 22 mg. - ANS*A: 19 mg.* Reason: Diltiazem hydrochloride (Cardizem) is a calcium channel blocking drug that is used to treat rapid ventricular rates associated with atrial fibrillation or atrial flutter. It can also be used after adenosine to treat refractory reentry supraventricular tachycardia in hemodynamically stable patients. The initial dose of diltiazem is 0.25 mg/kg IV over 2 minutes; the average initial dose is 15 to 20 mg. It may be repeated in 15 minutes in a dose of 0.35 mg/kg IV over 2 minutes; the average second dose is 20 to 25 mg. A 165- pound patient weighs 75 kg. Therefore, the initial dose of diltiazem for a patient of this weight would be 18.75 mg (approximately 19 mg), and the second dose would be 26.25 mg (approximately 26 mg). The initial dose of diltiazem for a 165-pound patient is approximately: - ANS19. Diltiazem hydrochloride (Cardizem) is a calcium channel blocking drug that is used to treat rapid ventricular rates associated with atrial fibrillation or atrial flutter. It can also be used after adenosine to treat refractory reentry supraventricular tachycardia in hemodynamically stable patients. The initial dose of diltiazem is 0.25 mg/kg IV over 2 minutes; the average initial dose is 15 to 20 mg. It may be repeated in 15 minutes in a dose of 0.35 mg/kg IV over 2 minutes; the average second dose is 20 to 25 mg. A 165- pound patient weighs 75 kg. Therefore, the initial dose of diltiazem for a patient of this weight would be 18.75 mg (approximately 19 mg), and the second dose would be 26.25 mg (approximately 26 mg). This type of seizure is described as an abrupt loss of muscle tone, sudden collapse and is sometimes known as a "drop attack" - ANSAtonic seizure This type of seizure is described as brief muscle contractions that usually occur at the same time on both sides of the body - ANSMyoclonic seizures thrill - ANSvibration the occurs frequently and remains constant To be classified as an organ it must have Howe many layers of tissue - ANS2 Tonsillar herniation - ANSWhen rising ICP causes portions of the brain to herniate through the foreman magnum Treating a patient who overdosed on a tricyclic antidepressant drug - ANSPer medical direction, sodium bicarb may begin to reverse cardiac toxicity. Airway, ventilatory and circulatory support; IV access, ECG monitoring. Treatment for a PT with GI bleeding includes - ANS2 L fluid bolus infusion in adults, 20 ml/kg in children. High flow 02 and pneumatic anti-shock garments should be considered Treatment of a delusional patient - ANSTreatment is aimed at correcting the underlying physical disorder to reduce anxiety. Sedatives may be required to manage the PT. Treatment of a headache - ANSTreatment in the prehospital setting is mostly supportive however a full history should be obtained and a full neurological exam should be performed. Most headaches can be managed with analgesics Treatment of a PT in A Fib - ANSadministration of a calcium channel blocker such as cardizem or a beta blocker Treatment of anaphylaxis - ANSGive Epinephrine to all patients with clinical signs of shock, airway swelling or difficulty breathing. Treatment of croup - ANSAdministration of cool mist or humidified or nebulized oxygen Treatment of organophosphate poisoning - ANSThe drugs currently used as antidotes include, atropine, pralidoxime chloride and diazepam or lorazepam. Treatment of polymorphic v tach - ANSUnsynchronized shocks(defibrillation) and magnesium sulfate Treatment of right ventricular failure - ANSAdministration of a 250 mL IV bolus over 5- 10 minutes Treatment of stable monomorphic V Tach - ANSProcainamide, amiodarone Treatment of unstable V Tach - ANSImmediate synchronized cardioversion at 100 J Treatment of ventricular fibrillation - ANSBLS, IV/IO, Defibrillation, Epi/vasopressin, amio/lido Tricuspid valve - ANSSeparates the right atrium and the right ventricle tunica adventitia - ANSprotective outer layer of fibrous tissue that provides blood vessels with the strength needed to withstand high pressure against their walls tunica intima - ANSinnermost layer of blood vessels; smooth inner lining that is only one cell thick tunica media - ANSmiddle layer of elastic fibers and muscle that gives strength and contractility to blood vessels. Much thicker and stronger in arteries compared to veins Unlike a second-degree AV block type I, a second-degree AV block type II is characterized by: A: a progressive lengthening of the P-R interval. B: dissociation of the P waves and QRS complexes. C: consistent P-R intervals following conducted P waves. D: a ventricular rate that is less than 50 beats/min. - ANS*C: consistent P-R intervals following conducted P waves* Reason: A second-degree AV block Mobitz Type II (classic second-degree AV block) is characterized by more P waves than QRS complexes. However, the P-R intervals of the conducted complexes (P waves that are followed by a QRS complex)—whether shortened, normal, or prolonged—are consistent. By contrast, a second-degree AV block Mobitz Type I (Wenkebach) is characterized by a progressive lengthening of the P-R interval until a P wave is blocked (not followed by a QRS complex). The ventricular rate of a second-degree AV block may be normal or slow. Dissociation of the P waves and QRS complexes is characteristic of a third-degree (complete) AV block. Unlike a second-degree AV block type I, a second-degree AV block type II is characterized by: - ANSConsistent P-R intervals following conducted P waves. A second-degree AV block Mobitz Type II (classic second-degree AV block) is characterized by more P waves than QRS complexes. However, the P-R intervals of the conducted complexes (P waves that are followed by a QRS complex)—whether shortened, normal, or prolonged—are consistent. By contrast, a second-degree AV block Mobitz Type I (Wenkebach) is characterized by a progressive lengthening of the P-R interval until a P wave is blocked (not followed by a QRS complex). The ventricular rate of a second-degree AV block may be normal or slow. Dissociation of the P waves and QRS complexes is characteristic of a third-degree (complete) AV block. Using a macrodrip drop set how many drops is 1ml? - ANS10 drops Using a microdrip set how many drops is 1ml? - ANS60 drops Vasopressin - ANS-Stimulation of smooth muscle receptors -Potent vasoconstrictor when given in high doses -Alternative vasopressor to the first or second dose of epinephrine in cardiac arrest, alternative to epinephrine in asystole, PEA Vasopressin dose - ANS40 U veins - ANSvessels that carry blood to the heart; generally deoxygenated blood (except the pulmonary veins). The largest are the inferior and superior vena cava ventricular contraction - ANSAV valves close; two ventricles contract and the semilunar valves are forced open. Blood squeezed out of the right ventricle moves forward into the pulmonary arteries and blood from the left ventricle is pushed through the aorta. Systole takes half the time as ventricular filling (~.28seconds) Ventricular ejection fraction is defined as the: A: amount of blood pumped out from either ventricle per contraction. B: volume of blood pumped into the left ventricle from the left atrium. C: percentage of blood in the ventricle pumped out during a contraction. D: amount of blood pumped from either ventricle each minute. - ANS*C: percentage of blood in the ventricle pumped out during a contraction* Reason: Ejection fraction (EF) is the percentage of blood that is pumped from the ventricle per contraction. The total volume of blood pumped out of the ventricle per contraction is called the stroke volume (SV). If the ventricle contains 100 mL of blood before a contraction, but only ejects 55 mL when it contracts (SV), the ejection fraction is 55% (100 mL × 0.55 = 55 mL). Ejection fraction should be at least 65% in the adult. Cardiac output (CO) is the volume of blood ejected from the left ventricle each minute, and is calculated by multiplying the stroke volume by the heart rate; in the adult, this is typically 5 to 6 L/min. Ventricular ejection fraction is defined as the: A: percentage of blood in the ventricle pumped out during a contraction. B: volume of blood pumped into the left ventricle from the left atrium. C: amount of blood pumped out from either ventricle per contraction. A: Systemic venous pooling of blood and increased afterload B: Decreased venous capacitance and increased inotropy C: Increased venous capacitance and decreased preload D: Increased cardiac inotropy and increased cardiac output - ANS*C: Increased venous capacitance and decreased preload* Reason: In patients with cardiogenic pulmonary edema (ie, congestive heart failure [CHF]), morphine sulfate causes systemic pooling of blood, which increases venous capacitance and decreases preload (the volume of blood returned to the heart). The net effect is to minimize the volume of fluid that accumulates in the lungs. Note that morphine is not a diuretic and will not remove fluid from the body. This is accomplished by administering furosemide (Lasix), which may be considered for patients with CHF and pulmonary edema. What are the therapeutic effects of morphine sulfate when administered to a patient with cardiogenic pulmonary edema? - ANSIncreased venous capacitance and decreased preload. In patients with cardiogenic pulmonary edema (ie, congestive heart failure [CHF]), morphine sulfate causes systemic pooling of blood, which increases venous capacitance and decreases preload (the volume of blood returned to the heart). The net effect is to minimize the volume of fluid that accumulates in the lungs. Note that morphine is not a diuretic and will not remove fluid from the body. This is accomplished by administering furosemide (Lasix), which may be considered for patients with CHF and pulmonary edema. What cartilage is an important visual landmark for endotracheal intubation - ANSArytenoid What ion is responsible for contraction of the myocardium? - ANSCalcium What ion is responsible for depolarization of the myocardium? - ANSSodium What ion is responsible for repolarization of the myocardium - ANSPotassium What is a delivery called that is delivered within the first 3 hours of labor - ANSPrecipitous delivery What is the cause of carpopedal spasms - ANSHyperventilation What is the correct initial dose and rate of administration of amiodarone for a patient with refractory ventricular fibrillation? A: 150 mg via rapid IV or IO push B: 300 mg via rapid IV or IO push C: 150 mg given over 10 minutes D: 300 mg given over 10 minutes - ANS*B: 300 mg via rapid IV or IO push* Reason: The correct initial dose and rate of administration of amiodarone for a patient with refractory ventricular fibrillation or pulseless ventricular tachycardia is 300 mg rapid IV or IO push. You may repeat amiodarone one time in 5 minutes at a dose of 150 mg rapid IV or IO push. For patients with hemodynamically stable narrow or wide-complex tachycardias, the correct dose and rate of administration for amiodarone is 150 mg given over 10 minutes. What is the correct initial dose and rate of administration of amiodarone for a patient with refractory ventricular fibrillation? - ANS300 mg via rapid IV or IO push. The correct initial dose and rate of administration of amiodarone for a patient with refractory ventricular fibrillation or pulseless ventricular tachycardia is 300 mg rapid IV or IO push. You may repeat amiodarone one time in 5 minutes at a dose of 150 mg rapid IV or IO push. For patients with hemodynamically stable narrow or wide-complex tachycardias, the correct dose and rate of administration for amiodarone is 150 mg given over 10 minutes. What is the most important extrauterine function? - ANSRespiration What is the PR interval in a NSR - ANS.12-.20 What is the ratio of fluid replacement to fluid loss in shock patients? - ANS3:1 What is the therapeutic effect of aspirin when administered to a patient experiencing an acute coronary syndrome (ACS)? A: Decreased thromboxane A2 production, which inhibits platelet aggregation B: Destruction of platelets by increasing thromboxane A2 production C: Destruction of a blood clot in a coronary artery by destroying fibrin D: Decreased platelet production and coronary artery vasoconstriction - ANS*A: Decreased thromboxane A2 production, which inhibits platelet aggregation* Reason: Thromboxane A2 is produced by activated platelets. It is a potent vasoconstrictor, it stimulates activation of new platelets, and it increases platelet aggregation. Aspirin (acetylsalicylic acid [ASA]) blocks the production of thromboxane A2, which inhibits vasoconstriction, inhibits activation of new platelets, and inhibits platelet aggregation (ie, it makes the platelets less "sticky"). Aspirin does not destroy a clot in a coronary artery —it prevents it from getting larger. Furthermore, by inhibiting local coronary vasoconstriction, it may enhance blood flow around the clot. Fibrinolytic agents (ie, alteplase [Activase], streptokinase [Streptase], tenecteplase [TNKase]) convert the body's own clot-dissolving enzyme from its inactive form, plasminogen, to its active form, plasmin. Plasmin then destroys the fibrin matrix of the clot—hence the term "fibrinolysis." What is the therapeutic effect of aspirin when administered to a patient experiencing an acute coronary syndrome (ACS)? - ANSDecreased thromboxane A2 production, which inhibits platelet aggregation. Thromboxane A2 is produced by activated platelets. It is a potent vasoconstrictor, it stimulates activation of new platelets, and it increases platelet aggregation. Aspirin (acetylsalicylic acid [ASA]) blocks the production of thromboxane A2, which inhibits vasoconstriction, inhibits activation of new platelets, and inhibits platelet aggregation (ie, it makes the platelets less "sticky"). Aspirin does not destroy a clot in a coronary artery —it prevents it from getting larger. Furthermore, by inhibiting local coronary vasoconstriction, it may enhance blood flow around the clot. Fibrinolytic agents (ie, alteplase [Activase], streptokinase [Streptase], tenecteplase [TNKase]) convert the body's own clot-dissolving enzyme from its inactive form, plasminogen, to its active form, plasmin. Plasmin then destroys the fibrin matrix of the clot—hence the term "fibrinolysis." What occurs at the beginning of ventricular contraction? A: Increased ventricular pressure causes the ventricular walls to stretch. B: The pulmonic and aortic valves close and the tricuspid and mitral valves open. C: The atrioventricular valves close and the semilunar valves are forced open. D: Additional blood fills the ventricles secondary to atrial kick. - ANS*C: The atrioventricular valves close and the semilunar valves are forced open* Reason: As ventricular contraction begins, the atrioventricular valves (tricuspid and mitral) close and the semilunar valves (pulmonic and aortic) are forced open. As a result, blood moves from the right ventricle through the pulmonary arteries and from the left ventricle through the aorta and into the systemic circulation. The majority of ventricular filling occurs by gravity. Atrial kick is the volume of blood that the atria contribute to ventricular filling; this occurs before ventricular contraction. Increased pressure within the myocardium (ie, increased blood volume) causes stretching of the myocardial walls, thus increasing the force of its contraction (Starling effect); this process precedes ventricular contraction. What occurs at the beginning of ventricular contraction? - ANSThe atrioventricular valves close and the semilunar valves are forced open. As ventricular contraction begins, the atrioventricular valves (tricuspid and mitral) close and the semilunar valves (pulmonic and aortic) are forced open. As a result, blood moves from the right ventricle through the pulmonary arteries and from the left ventricle through the aorta and into the systemic circulation. The majority of ventricular filling occurs by gravity. Atrial kick is the volume of blood that the atria contribute to ventricular patients with a heart rate less than 150 beats/min. Tachycardia in the patient with myocardial ischemia is NOT good; it increases myocardial oxygen demand and consumption, which can exacerbate ischemia. Calcium channel blockers (eg, diltiazem [Cardizem]) are commonly used for ventricular rate control in patients with atrial fibrillation or atrial flutter. Synchronized cardioversion is indicated for patients with hemodynamic compromise secondary to supraventricular tachycardia (narrow complex; heart rate > 150 beats/min) and ventricular tachycardia (wide complex; rate > 100 beats/min [often > 200 beats/min]). When assessing a patient with sinus tachycardia at a rate of 135 beats/min, you should recall that: - ANSRate-related symptoms are uncommon in patients with a heart rate less than 150 beats/min. Sinus tachycardia in the adult—that is, a heart rate less than 150 beats/min—is usually a manifestation of an underlying problem, such as hypovolemia or hypoxia. Therefore, the treatment for sinus tachycardia should focus on treating the underlying cause (ie, fluid boluses, oxygen). Rate-related hemodynamic compromise is uncommon in patients with a heart rate less than 150 beats/min. Tachycardia in the patient with myocardial ischemia is NOT good; it increases myocardial oxygen demand and consumption, which can exacerbate ischemia. Calcium channel blockers (eg, diltiazem [Cardizem]) are commonly used for ventricular rate control in patients with atrial fibrillation or atrial flutter. Synchronized cardioversion is indicated for patients with hemodynamic compromise secondary to supraventricular tachycardia (narrow complex; heart rate > 150 beats/min) and ventricular tachycardia (wide complex; rate > 100 beats/min [often > 200 beats/min]). When assessing a patient with suspected cardiac-related chest pain, which of the following questions would be MOST appropriate to ask? A: Does the pain move to your arms? B: Can you describe the quality of the pain? C: Were you at rest when the pain began? D: Is the pain crushing or dull in nature? - ANS*B: Can you describe the quality of the pain?* Reason: Patient assessment involves simple questioning techniques. You should ask open- ended questions, whenever possible; this is especially true when determining the onset and quality of a patient's pain. Asking a leading question, such as "Do you have sharp chest pain?" will often lead the patient to say "yes," even though that is not the true quality of his or her pain. Allow the patient to use his or her own words when describing symptoms. When assessing a patient with suspected cardiac-related chest pain, which of the following questions would be MOST appropriate to ask? - ANSCan you describe the quality of the pain? Patient assessment involves simple questioning techniques. You should ask open- ended questions, whenever possible; this is especially true when determining the onset and quality of a patient's pain. Asking a leading question, such as "Do you have sharp chest pain?" will often lead the patient to say "yes," even though that is not the true quality of his or her pain. Allow the patient to use his or her own words when describing symptoms. When assessing a patient's pulse, you note that it is fast and has an irregularly irregular pattern. On the basis of these findings, which of the following cardiac rhythms would MOST likely be seen on the cardiac monitor? A: Supraventricular tachycardia B: Ventricular tachycardia C: Uncontrolled atrial fibrillation D: Second-degree AV block type 1 - ANS*C: Uncontrolled atrial fibrillation* Reason: Of the cardiac rhythms listed, atrial fibrillation (A-Fib) is the only one that is irregularly irregular. In fact, A-Fib is never seen as a regular rhythm. At a rate of less than 100 beats/min, A-Fib is said to be controlled. Uncontrolled A-Fib, or A-Fib with a rapid ventricular rate (RVR), occurs when the ventricular rate exceeds 100 beats/min. Second-degree AV block type I has a pattern that is regularly irregular; the P-R interval progressively lengthens until a P wave is blocked. Ventricular tachycardia (V-Tach) and supraventricular tachycardia (SVT) are typically regular rhythms. When assessing a patient's pulse, you note that it is fast and has an irregularly irregular pattern. On the basis of these findings, which of the following cardiac rhythms would MOST likely be seen on the cardiac monitor? - ANSUncontrolled atrial fibrillation. Of the cardiac rhythms listed, atrial fibrillation (A-Fib) is the only one that is irregularly irregular. In fact, A-Fib is never seen as a regular rhythm. At a rate of less than 100 beats/min, A-Fib is said to be controlled. Uncontrolled A-Fib, or A-Fib with a rapid ventricular rate (RVR), occurs when the ventricular rate exceeds 100 beats/min. Second-degree AV block type I has a pattern that is regularly irregular; the P-R interval progressively lengthens until a P wave is blocked. Ventricular tachycardia (V-Tach) and supraventricular tachycardia (SVT) are typically regular rhythms. When assessing lead II in a patient with a heart rate of 70 beats/min, the Q-T interval is considered prolonged if it is: A: twice the width of the QRS complex. B: three times the length of the P-R interval. C: greater than one half of the R-R interval. D: consistently greater than 0.20 seconds. - ANS*C: greater than one half of the R-R interval.* Reason: The Q-T interval represents the time from the beginning of ventricular depolarization to the end of ventricular repolarization, and is measured from the start of the QRS complex to the end of the T wave. In a patient with a heart rate between 60 and 100 beats/min, the Q-T interval in lead II is considered to be prolonged if it is greater than one half the distance between any two R waves (R-R interval). If the Q-T interval is prolonged, the patient is at increased risk for developing a lethal dysrhythmia; an electrical impulse may fire during the relative refractory period (downslope of the T-wave), resulting in monomorphic or polymorphic ventricular tachycardia (with or without a pulse) or ventricular fibrillation. If lead II suggests Q-T prolongation, a 12-lead ECG should be obtained to quantify this finding. In a normocardic patient (heart rate of 60 to 100 beats/min), the corrected Q-T interval (QTc) should range between 0.36 and 0.44 seconds (360 to 440 milliseconds) on the 12-lead ECG. The Q-T interval is corrected based on the patient's heart rate. The faster the heart rate, the narrower the Q-T interval; the slower the heart rate, the wider the Q-T interval. When assessing lead II in a patient with a heart rate of 70 beats/min, the Q-T interval is considered prolonged if it is: - ANSGreater than one half of the R-R interval. The Q-T interval represents the time from the beginning of ventricular depolarization to the end of ventricular repolarization, and is measured from the start of the QRS complex to the end of the T wave. In a patient with a heart rate between 60 and 100 beats/min, the Q-T interval in lead II is considered to be prolonged if it is greater than one half the distance between any two R waves (R-R interval). If the Q-T interval is prolonged, the patient is at increased risk for developing a lethal dysrhythmia; an electrical impulse may fire during the relative refractory period (downslope of the T-wave), resulting in monomorphic or polymorphic ventricular tachycardia (with or without a pulse) or ventricular fibrillation. If lead II suggests Q-T prolongation, a 12-lead ECG should be obtained to quantify this finding. In a normocardic patient (heart rate of 60 to 100 beats/min), the corrected Q-T interval (QTc) should range between 0.36 and 0.44 seconds (360 to 440 milliseconds) on the 12-lead ECG. The Q-T interval is corrected based on the patient's heart rate. The faster the heart rate, the narrower the Q-T interval; the slower the heart rate, the wider the Q-T interval. When attempting transcutaneous cardiac pacing (TCP), you will know that electrical capture has been achieved when: A: the patient's inherent heart rate spontaneously increases. B: each pacemaker spike is followed by a wide QRS complex. C: the milliamp setting is at least 40 and the patient is in pain. D: you see an increase in the number of narrow QRS complexes. - ANS*B: each pacemaker spike is followed by a wide QRS complex* Reason: Transcutaneous cardiac pacing (TCP) involves passing small, repetitive electrical currents through the patient's skin (transcutaneous) across the heart between one transcutaneous cardiac pacing (TCP) without delay. If the patient is in a second-degree type II or third-degree AV block, TCP is the first-line treatment. Atropine and TCP- refractory bradycardia may require a sympathomimetic infusion, such as epinephrine or dopamine. The body's normal physiologic response to hypovolemia is tachycardia, not bradycardia. Therefore, fluid boluses are not the initial treatment for the hypotensive, bradycardic patient. In fact, they may cause further harm to the patient. With a slow heart rate and decreased cardiac output, a sudden increase in preload may result in acute pulmonary edema. After stabilizing the patient's heart rate and improving perfusion, obtain a 12-lead ECG to assess for signs of acute myocardial ischemia or injury. Where is the point of maximal impulse (PMI) located in most people? A: Directly over the sternum, approximately 1" to the left of the angle of Louis B: Left anterior chest, in the midclavicular line, at the fifth intercostal space C: Left anterior chest, on the left sternal border, at the fourth intercostal space D: Left anterolateral chest, in the midaxillary line, at the fifth intercostal space - ANS*B: Left anterior chest, in the midclavicular line, at the fifth intercostal space* Where is the point of maximal impulse (PMI) located in most people? - ANSLeft anterior chest, in the midclavicular line, at the fifth intercostal space. On visualization of the chest, you may be able to see the apical thrust, or point of maximal impulse (PMI). The PMI is normally located on the left anterior part of the chest, in the midclavicular line, at the fifth intercostal space. This thrust occurs when the apex of the heart rotates forward during systole, gently beating against the chest wall and producing a visible pulsation. Where to approach a helicopter - ANSFrom the front Which of the following 12-lead ECG findings signifies a left bundle branch block? A: QRS duration of 122 ms; terminal S wave in lead V6 B: QRS duration of 126 ms; terminal S wave in lead aVL C: QRS duration of 128 ms; terminal R wave in lead V1 D: QRS duration of 124 ms; terminal S wave in lead V1 - ANS*D: QRS duration of 124 ms; terminal S wave in lead V1* Reason: A QRS duration of greater than 120 ms (0.12 seconds [3 small boxes]) signifies an intraventricular conduction delay (IVCD), such as a bundle branch block. A left bundle branch block (LBBB) is characterized by a QRS duration of greater than 120 ms and a terminal S wave in lead V1 (the second half of the QRS complex terminates in an S wave); terminal R waves are seen in leads I, aVL, and V6. A right bundle branch block (RBBB) is characterized by a QRS duration of greater than 120 ms and a terminal R wave in lead V1 (the second half of the QRS complex terminates in an R wave); terminal S waves are seen in leads I, aVL, and V6. Which of the following 12-lead ECG findings signifies a left bundle branch block? - ANSQRS duration of 124 ms; terminal S wave in lead V1 . A QRS duration of greater than 120 ms (0.12 seconds [3 small boxes]) signifies an intraventricular conduction delay (IVCD), such as a bundle branch block. A left bundle branch block (LBBB) is characterized by a QRS duration of greater than 120 ms and a terminal S wave in lead V1 (the second half of the QRS complex terminates in an S wave); terminal R waves are seen in leads I, aVL, and V6. A right bundle branch block (RBBB) is characterized by a QRS duration of greater than 120 ms and a terminal R wave in lead V1 (the second half of the QRS complex terminates in an R wave); terminal S waves are seen in leads I, aVL, and V6. Which of the following cardiac rhythms is associated with bradycardia, and is characterized by regular R-R intervals and a greater ratio of P waves to QRS complexes? A: First-degree AV block B: Second-degree AV block type I C: Second-degree AV block type II D: Third-degree AV block - ANS*D: Third-degree AV block* Reason: Third-degree AV block is caused by a complete block at the AV node. The SA node initiates impulses as usual; however, when they reach the AV node, they are blocked. Resultantly, the ventricles receive no electrical stimulus from the atria, so they initiate their own impulses, although at a much slower rate. On the ECG, this manifests as a bradycardic rhythm with more P waves than QRS complexes. The P-P intervals are regular (some P waves may not be visible because they are buried in a QRS complex), as are the R-R intervals; however, no relationship exists between a given P wave and QRS complex. Second-degree AV block type I (Wenkebach) is caused by a progressive delay at the AV node until an impulse is blocked from entering the ventricles. On the ECG, this manifests as a progressively lengthening P-R interval until a P wave is blocked (not followed by a QRS complex). At this point, the R-R interval becomes irregular, and the presence of this lone P wave increases the ratio of P waves to QRS complexes. Second-degree AV block type I may or may not be associated with bradycardia. Second-degree AV block type II is caused by an intermittent block at the AV node; it occurs when atrial impulses are not conducted to the ventricles. Unlike a second-degree AV block type I, however, a type II block is characterized by consistent P-R intervals of the P waves that are conducted. First-degree AV block is an abnormal delay at the AV node; on the ECG, this manifests with PR intervals greater than 0.20 seconds (120 ms) in duration. In first-degree AV block, all of the atrial impulses are conducted through the AV node and into the ventricles. Which of the following cardiac rhythms is associated with bradycardia, and is characterized by regular R-R intervals and a greater ratio of P waves to QRS complexes? - ANS3rd degree AV block. Third-degree AV block is caused by a complete block at the AV node. The SA node initiates impulses as usual; however, when they reach the AV node, they are blocked. Resultantly, the ventricles receive no electrical stimulus from the atria, so they initiate their own impulses, although at a much slower rate. On the ECG, this manifests as a bradycardic rhythm with more P waves than QRS complexes. The P-P intervals are regular (some P waves may not be visible because they are buried in a QRS complex), as are the R-R intervals; however, no relationship exists between a given P wave and QRS complex. Second-degree AV block type I (Wenkebach) is caused by a progressive delay at the AV node until an impulse is blocked from entering the ventricles. On the ECG, this manifests as a progressively lengthening P-R interval until a P wave is blocked (not followed by a QRS complex). At this point, the R-R interval becomes irregular, and the presence of this lone P wave increases the ratio of P waves to QRS complexes. Second-degree AV block type I may or may not be associated with bradycardia. Second-degree AV block type II is caused by an intermittent block at the AV node; it occurs when atrial impulses are not conducted to the ventricles. Unlike a second-degree AV block type I, however, a type II block is characterized by consistent P-R intervals of the P waves that are conducted. First-degree AV block is an abnormal delay at the AV node; on the ECG, this manifests with PR intervals greater than 0.20 seconds (120 ms) in duration. In first-degree AV block, all of the atrial impulses are conducted through the AV node and into the ventricles. Which of the following causes of pulseless electrical activity (PEA) would be the MOST likely to respond to immediate treatment in the prehospital setting? A: Lactic acidosis B: Hypokalemia C: Hypovolemia D: Drug overdose - ANS*C: Hypovolemia* Reason: Hypovolemia is the most easily correctable cause of PEA, provided that immediate treatment is given in the prehospital setting. In addition to CPR, airway management, and epinephrine, fluid boluses are repeatedly given, followed by a reassessment of the patient's condition. Remember, myocardial contraction is dependent on electricity and pressure. This pressure is caused as blood fills the heart. If there is no blood, the heart will not pump, even though electrical activity continues. Drug overdose is the underlying cause of asystole that would most likely respond to immediate prehospital treatment, especially in younger patients. Hypokalemia is treated with potassium chloride, which is not administered in the prehospital setting. Lactic acidosis is treated with effective ventilation first, and then sodium bicarbonate if local protocol permits. While sodium bicarbonate can be given in the prehospital setting, paramedics do not have the ability to quantify the pH or bicarbonate level of the patient's blood; this requires arterial blood gas analysis. Which of the following causes of pulseless electrical activity (PEA) would be the MOST likely to respond to immediate treatment in the prehospital setting? C: Gradual onset of chest pressure that increases in severity over time, hypotension, tachycardia, bilaterally weak radial pulses D: Sudden onset of lower back pain that radiates to the groin, urge to defecate, pain is constant and moderate in severity - ANS*A: Acute tearing pain in between the scapulae, blood pressure discrepancy between arms, maximal pain severity from the onset* Reason: Aortic dissection occurs when the layers of the aorta undergo destructive changes, resulting in an aneurysm (weakening and ballooning of the arterial wall). In dissection of the ascending aorta, the patient typically experiences an acute onset of ripping, tearing, or stabbing pain in the anterior chest or in between the scapulae. In some patients, it may be difficult to differentiate the pain of acute aortic dissection from that of acute myocardial infarction (AMI); however, a number of distinctive features may help. The pain of an AMI is often preceded by prodromal symptoms (eg, nausea, weakness, sweating). Although pain from an AMI is acute, it gradually intensifies over time and is typically described as a squeezing or pressure sensation. By contrast, the pain of aortic dissection is acute, is of maximal intensity from the onset, and is usually described as a ripping, tearing, or stabbing feeling. Other signs and symptoms depend on the extent and location of the dissection. In dissections of the ascending aorta, one or more of the vessels of the aortic arch may be compromised. Disruption of blood flow through the innominate artery, for example, is likely to produce a difference in blood pressure between the arms. The onset and pain characteristics of abdominal aortic dissection are similar to those of ascending aortic dissection; however, the pain typically begins in the abdomen or lower back. Pulse deficits in the femoral arteries may be present, and if the aneurysm is leaking blood into the retroperitoneal space, the patient may complain of an urge to defecate and exhibit signs of shock. Which of the following clinical presentations is MOST consistent with dissection of the ascending aorta? - ANSAcute tearing pain in between the scapulae, blood pressure discrepancy between arms, maximal pain severity from the onset. Aortic dissection occurs when the layers of the aorta undergo destructive changes, resulting in an aneurysm (weakening and ballooning of the arterial wall). In dissection of the ascending aorta, the patient typically experiences an acute onset of ripping, tearing, or stabbing pain in the anterior chest or in between the scapulae. In some patients, it may be difficult to differentiate the pain of acute aortic dissection from that of acute myocardial infarction (AMI); however, a number of distinctive features may help. The pain of an AMI is often preceded by prodromal symptoms (eg, nausea, weakness, sweating). Although pain from an AMI is acute, it gradually intensifies over time and is typically described as a squeezing or pressure sensation. By contrast, the pain of aortic dissection is acute, is of maximal intensity from the onset, and is usually described as a ripping, tearing, or stabbing feeling. Other signs and symptoms depend on the extent and location of the dissection. In dissections of the ascending aorta, one or more of the vessels of the aortic arch may be compromised. Disruption of blood flow through the innominate artery, for example, is likely to produce a difference in blood pressure between the arms. The onset and pain characteristics of abdominal aortic dissection are similar to those of ascending aortic dissection; however, the pain typically begins in the abdomen or lower back. Pulse deficits in the femoral arteries may be present, and if the aneurysm is leaking blood into the retroperitoneal space, the patient may complain of an urge to defecate and exhibit signs of shock. Which of the following ECG findings indicates a pathologic delay at the AV node? A: P-R interval of 0.28 seconds B: P waves of varying morphologies C: P-R interval less than 0.12 seconds D: QRS complex of 0.16 seconds - ANS*A: P-R interval of 0.28 seconds* Reason: Normally, there is a physiologic delay of an impulse at the AV node that allows the atria to empty into the ventricles. On the ECG, this manifests as a P-R interval—the period of time that includes atrial depolarization and the delay at the AV node—that is between 0.12 and 0.20 seconds (120 to 200 ms). A pathologic delay at the AV node, such as what occurs with a first-degree AV block, would manifest with a P-R interval that is greater than 0.12 seconds (120 ms) in duration. By contrast, A P-R interval that is less than 0.12 seconds indicates that an impulse is traversing the AV node too fast or is bypassing it altogether, such as what occurs with Wolff-Parkinson-White (WPW) syndrome, a preexcitation syndrome in which the electrical impulse follows accessory pathways around the AV node (bundle of Kent) and prematurely depolarizes the ventricles. A wide (> 0.12 seconds [120 ms]) QRS complex indicates an intraventricular conduction delay, such as a bundle branch block. P waves that vary in morphology (appearance) indicate more than one atrial pacemaker site; an example of this is an ectopic atrial rhythm. Which of the following ECG findings indicates a pathologic delay at the AV node? - ANSP-R interval of 0.28 seconds. Normally, there is a physiologic delay of an impulse at the AV node that allows the atria to empty into the ventricles. On the ECG, this manifests as a P-R interval—the period of time that includes atrial depolarization and the delay at the AV node—that is between 0.12 and 0.20 seconds (120 to 200 ms). A pathologic delay at the AV node, such as what occurs with a first-degree AV block, would manifest with a P-R interval that is greater than 0.12 seconds (120 ms) in duration. By contrast, A P-R interval that is less than 0.12 seconds indicates that an impulse is traversing the AV node too fast or is bypassing it altogether, such as what occurs with Wolff-Parkinson-White (WPW) syndrome, a preexcitation syndrome in which the electrical impulse follows accessory pathways around the AV node (bundle of Kent) and prematurely depolarizes the ventricles. A wide (> 0.12 seconds [120 ms]) QRS complex indicates an intraventricular conduction delay, such as a bundle branch block. P waves that vary in morphology (appearance) indicate more than one atrial pacemaker site; an example of this is an ectopic atrial rhythm. Which of the following ECG lead configurations is correct? A: To assess lead II, place the negative lead on the right arm and the positive lead on the left leg. B: To assess lead III, place the negative lead on the right arm and the positive lead on the left leg. C: To assess lead III, place the negative lead on the left leg and the positive lead on the right arm. D: To assess lead I, place the positive lead on the right arm and the negative lead on the left arm. - ANS*A: To assess lead II, place the negative lead on the right arm and the positive lead on the left leg.* Reason: According to the Einthoven triangle, lead I is assessed by placing the negative (white) lead on the right arm and the positive (red) lead on the left arm. Lead II is assessed by placing the negative lead on the right arm and the positive lead on the left leg. Lead III is assessed by placing the negative lead on the left arm and the positive lead on the left leg. Which of the following ECG lead configurations is correct? - ANSTo assess lead II, place the negative lead on the right arm and the positive lead on the left leg. According to the Einthoven triangle, lead I is assessed by placing the negative (white) lead on the right arm and the positive (red) lead on the left arm. Lead II is assessed by placing the negative lead on the right arm and the positive lead on the left leg. Lead III is assessed by placing the negative lead on the left arm and the positive lead on the left leg. Which of the following electrolytes moves slowly into the cardiac cell and maintains the depolarized state of the cell membrane? A: Magnesium B: Calcium C: Sodium D: Potassium - ANS*B: Calcium* Reason: The process of depolarization begins as sodium ions rush into the cell. At the same time, calcium ions enter the cell—albeit more slowly and through specialized channels —to help maintain the depolarized state of the cell membrane and to supply calcium ions for contraction of cardiac muscle tissue. During repolarization, the sodium and calcium channels close, thus stopping the rapid influx of these ions. Then, special potassium channels open, allowing potassium ions to rapidly exit the cell. This helps restore the inside of the cell to its negative charge; the proper electrolyte distribution is then reestablished by pumping sodium ions out of the cell and potassium ions back in. After the potassium channels close, the sodium-potassium pump helps move sodium and potassium ions back to their respective locations. For every three sodium ions the defibrillation decreases over time, especially if CPR is delayed. For each minute that V- Fib persists, the patient's chance of survival decreases by approximately 7% to 10%. Which of the following interventions has the greatest impact on patient survival from sudden cardiac arrest? - ANSEarly CPR and defibrillation. Early CPR and defibrillation are the two interventions that will have the greatest impact on patient survival from sudden cardiac arrest (SCA). Early, effective CPR maintains perfusion to the body's vital organs until defibrillation can be provided. The most common initial cardiac rhythm observed during SCA is ventricular fibrillation (V-Fib). Early defibrillation, in conjunction with early CPR, greatly enhances the chance of establishing return of spontaneous circulation (ROSC). The probability of successful defibrillation decreases over time, especially if CPR is delayed. For each minute that V- Fib persists, the patient's chance of survival decreases by approximately 7% to 10%. Which of the following is an absolute contraindication for fibrinolytic therapy? A: Current use of anticoagulant medication B: Subdural hematoma 3 years ago C: BP of 170/100 mm Hg on presentation D: Ischemic stroke within the last 6 months - ANS*B: Subdural hematoma 3 years ago* Reason: According to current emergency cardiac care (ECC) guidelines, absolute contraindications for fibrinolytic therapy include ANY prior intracranial hemorrhage (ie, subdural, epidural, intracerebral hematoma); known structural cerebrovascular lesion (ie, arteriovenous malformation); known malignant intracranial tumor (primary or metastatic); ischemic stroke within the past 3 months, EXCEPT for acute ischemic stroke within the past 3 hours; suspected aortic dissection; active bleeding or bleeding disorders (except menses); and significant closed head trauma or facial trauma within the past 3 months. Relative contraindications (eg, the physician may deem fibrinolytic therapy appropriate under certain circumstances) include, a history of chronic, severe, poorly-controlled hypertension; severe uncontrolled hypertension on presentation (SBP > 180 mm Hg or DBP > 110 mm Hg); ischemic stroke greater than 3 months ago; dementia; traumatic or prolonged (> 10 minutes) CPR or major surgery within the past 3 weeks; recent (within 2 to 4 weeks) internal bleeding; noncompressible vascular punctures; pregnancy; prior exposure (> 5 days ago) or prior allergic reaction to streptokinase or anistreplase; active peptic ulcer; and current use of anticoagulants (ie, Coumadin). Which of the following is an absolute contraindication for fibrinolytic therapy? - ANSSubdural hematoma 3 years ago. According to current emergency cardiac care (ECC) guidelines, absolute contraindications for fibrinolytic therapy include ANY prior intracranial hemorrhage (ie, subdural, epidural, intracerebral hematoma); known structural cerebrovascular lesion (ie, arteriovenous malformation); known malignant intracranial tumor (primary or metastatic); ischemic stroke within the past 3 months, EXCEPT for acute ischemic stroke within the past 3 hours; suspected aortic dissection; active bleeding or bleeding disorders (except menses); and significant closed head trauma or facial trauma within the past 3 months. Relative contraindications (eg, the physician may deem fibrinolytic therapy appropriate under certain circumstances) include, a history of chronic, severe, poorly-controlled hypertension; severe uncontrolled hypertension on presentation (SBP > 180 mm Hg or DBP > 110 mm Hg); ischemic stroke greater than 3 months ago; dementia; traumatic or prolonged (> 10 minutes) CPR or major surgery within the past 3 weeks; recent (within 2 to 4 weeks) internal bleeding; noncompressible vascular punctures; pregnancy; prior exposure (> 5 days ago) or prior allergic reaction to streptokinase or anistreplase; active peptic ulcer; and current use of anticoagulants (ie, Coumadin). Which of the following pain descriptions is MOST consistent with a cardiac problem? A: Crushing B: Sharp C: Tearing D: Intermittent - ANS*A: Crushing* Reason: Chest pain of cardiac origin is most often described as crushing, dull, pressure, or as a feeling of heaviness or discomfort. The pain is typically constant, not intermittent, and is usually not palliated or exacerbated by movement. Bear in mind that these are typical pain descriptions. The paramedic should not rule out a cardiac problem if the patient describes the pain differently. Sharp (pleuritic) pain is often associated with conditions such as pleurisy, pulmonary embolism, or spontaneous pneumothorax. A tearing sensation should alert you to the possibility of acute aortic dissection. Which of the following pain descriptions is MOST consistent with a cardiac problem? - ANSCrushing. Chest pain of cardiac origin is most often described as crushing, dull, pressure, or as a feeling of heaviness or discomfort. The pain is typically constant, not intermittent, and is usually not palliated or exacerbated by movement. Bear in mind that these are typical pain descriptions. The paramedic should not rule out a cardiac problem if the patient describes the pain differently. Sharp (pleuritic) pain is often associated with conditions such as pleurisy, pulmonary embolism, or spontaneous pneumothorax. A tearing sensation should alert you to the possibility of acute aortic dissection. Which of the following represents the correct adult dosing regimen for adenosine? A: 12 mg, followed by 12 mg in 2 minutes if needed B: 6 mg, followed by 12 mg in 2 minutes if needed C: 36 mg, divided in 12 mg increments 2 minutes apart D: 6 mg, followed by 6 mg in 2 minutes if needed - ANS*B: 6 mg, followed by 12 mg in 2 minutes if needed* Reason: According to the 2010 guidelines for CPR and emergency cardiac care (ECC), the correct dosing regimen of adenosine for a hemodynamically stable patient with a narrow-complex tachycardia is 6 mg via rapid (over 1 to 3 seconds) IV push. If needed, adenosine can be repeated in 1 to 2 minutes in a dose of 12 mg rapid IV push. Which of the following represents the correct adult dosing regimen for adenosine? - ANS6 mg, followed by 12 mg in 2 minutes if needed. According to the 2010 guidelines for CPR and emergency cardiac care (ECC), the correct dosing regimen of adenosine for a hemodynamically stable patient with a narrow-complex tachycardia is 6 mg via rapid (over 1 to 3 seconds) IV push. If needed, adenosine can be repeated in 1 to 2 minutes in a dose of 12 mg rapid IV push. Which of the following represents the correct medication sequence when treating a patient with a suspected acute coronary syndrome? A: Oxygen, aspirin, morphine, and nitroglycerin B: Oxygen, nitroglycerin, aspirin, and morphine C: Oxygen, aspirin, nitroglycerin, and morphine D: Oxygen, morphine, aspirin, and nitroglycerin - ANS*C: Oxygen, aspirin, nitroglycerin, and morphine* Reason: The mnemonic "MONA" is used to help remember the medications given to patients who are experiencing an acute coronary syndrome (ACS). Although it does not represent the correct sequence in which the medications should be given, it is a useful mnemonic to remember. The appropriate sequence of medications is oxygen (as needed to maintain an SpO2 of greater than 94%), aspirin (160 to 325 mg), nitrogylcerin (0.4 mg up to 3 times), and morphine (2 to 4 mg) if the nitroglycerin does not relieve the chest pain. Pain relief is very important in patients experiencing ACS (eg, unstable angina or AMI) because it reduces anxiety and subsequent oxygen consumption and demand. Which of the following represents the correct medication sequence when treating a patient with a suspected acute coronary syndrome? - ANSOxygen, aspirin, nitroglycerin, and morphine. The mnemonic "MONA" is used to help remember the medications given to patients who are experiencing an acute coronary syndrome (ACS). Although it does not represent the correct sequence in which the medications should be given, it is a useful mnemonic to remember. The appropriate sequence of medications is oxygen (as needed to maintain an SpO2 of greater than 94%), aspirin (160 to 325 mg), nitrogylcerin (0.4 mg up to 3 times), and morphine (2 to 4 mg) if the nitroglycerin does not relieve the and commonly occurs as the result of right ventricular infarction (RVI). Treat the hypotensive patient with crystalloid fluid boluses (250 to 500 mL), which will increase preload and may improve contractility via the Starling effect. Vasodilators (ie, morphine, nitroglycerin) should not be administered to patients with RVF; they may induce or exacerbate hypotension. Which of the following statements regarding right ventricular failure (RVF) is correct? - ANSSacral and pedal edema are common signs of RVF. The most common cause of right ventricular failure (RVF) is left ventricular failure (LVF). When the left ventricle fails, blood backs up into the lungs and eventually into the pulmonary circulation, resulting in pulmonary hypertension. Because the right ventricle must work harder to overcome the increased resistance in the pulmonary circulation, it eventually fails as an effective forward pump. As a result, blood backs up into the systemic circulation, resulting in jugular venous distention, hepatomegaly (enlarged liver), and peripheral edema—especially to dependent areas of the body (eg, extremities, the sacrum in bedridden patients). In patients with severe RVF, total body edema (anasarca) may be present. Hypotension may be observed in patients with RVF, and commonly occurs as the result of right ventricular infarction (RVI). Treat the hypotensive patient with crystalloid fluid boluses (250 to 500 mL), which will increase preload and may improve contractility via the Starling effect. Vasodilators (ie, morphine, nitroglycerin) should not be administered to patients with RVF; they may induce or exacerbate hypotension. Which of the following statements regarding the use of vasopressin in cardiac arrest is correct? A: Vasopressin should be given every 3 to 5 minutes throughout the arrest B: Vasopressin is highly effective in treating pediatric cardiac arrest patients C: Vasopressin is superior to epinephrine and should be used when possible D: Vasopressin can be used to replace the first or second dose of epinephrine - ANS*D: Vasopressin can be used to replace the first or second dose of epinephrine* Reason: According to the 2010 guidelines for CPR and emergency cardiac care (ECC), vasopressin, in a one-time dose of 40 units, can be given to replace the first OR second dose of epinephrine for adult patients in cardiac arrest. There are no definitive data to support superiority of vasopressin over epinephrine. There are insufficient data to make a recommendation for or against the use of vasopressin in pediatric cardiac arrest. Which of the following statements regarding the use of vasopressin in cardiac arrest is correct? - ANSVasopressin can be used to replace the first or second dose of epinephrine. According to the 2010 guidelines for CPR and emergency cardiac care (ECC), vasopressin, in a one-time dose of 40 units, can be given to replace the first OR second dose of epinephrine for adult patients in cardiac arrest. There are no definitive data to support superiority of vasopressin over epinephrine. There are insufficient data to make a recommendation for or against the use of vasopressin in pediatric cardiac arrest. Which type of patient is most likely to experience HHNS? - ANSOlder patients with type 2 diabetes or patients with undiagnosed diabetes While assessing a middle-aged man who complains of nausea and weakness, he suddenly becomes unresponsive. The cardiac monitor displays the rhythm shown below. After determining that he is apneic and pulseless, you should: - ANSStart CPR and prepare to defibrillate. You witnessed your patient's deterioration to cardiac arrest, and he is now in ventricular fibrillation (V-Fib). You should immediately start CPR and defibrillate as soon as possible. Deliver a single shock with 360 monophasic joules or the equivalent biphasic setting, and immediately resume CPR (starting with chest compressions). Perform 5 cycles (about 2 minutes) of CPR and then reassess his cardiac rhythm. If V-Fib persists, defibrillate again and immediately resume CPR, starting with chest compressions. During CPR, establish vascular access (if not already done), and give 1 mg of epinephrine 1:10,000. After 2 minutes of CPR, reassess the patient's cardiac rhythm. If V-Fib persists, defibrillate again and immediately resume CPR, starting with chest compressions. It would then be appropriate to administer 300 mg of amiodarone. Synchronized cardioversion is indicated for patients with narrow or wide-complex tachycardias who are hemodynamically unstable but have a pulse. Wolff-Parkinson White - ANScharacterized by a short PR interval, widened QRS and a delta wave You and your team are attempting to resuscitate a 66-year-old man in cardiac arrest. The cardiac monitor reveals a slow, wide-complex rhythm. The patient has been successfully intubated and an IV line has been established. As CPR is ongoing, you should: A: give 40 units of vasopressin every 3 to 5 minutes. B: administer 10 mL of epinephrine 1:10,000 IV. C: ventilate the patient at a rate of 24 breaths/min. D: attempt transcutaneous pacing to increase the heart rate. - ANS*B: administer 10 mL of epinephrine 1:10,000 IV* Reason: The first drug given to any patient in cardiac arrest is epinephrine in a dose of 1 mg (10 mL of a 1:10,000 solution) via the IV or IO route. This dose should be repeated every 3 to 5 minutes. Alternatively, a one-time dose of vasopressin (40 units) can be given to replace the first or second dose of epinephrine, but not both. Do NOT hyperventilate the patient as doing so increases intrathoracic pressure and can impair venous return (preload) and cardiac output, which would decrease the effectiveness of chest compressions. After an advanced airway has been placed during cardiac arrest, deliver one breath every 6 to 8 seconds (8 to 10 breaths/min) and ensure that chest compressions are uninterrupted. There is presently no evidence to support the efficacy of transcutaneous cardiac pacing (TCP) in patients with bradycardic PEA or asystole. You and your team are attempting to resuscitate a 66-year-old man in cardiac arrest. The cardiac monitor reveals a slow, wide-complex rhythm. The patient has been successfully intubated and an IV line has been established. As CPR is ongoing, you should: - ANSAadminister 10 mL of epinephrine 1:10,000 IV. The first drug given to any patient in cardiac arrest is epinephrine in a dose of 1 mg (10 mL of a 1:10,000 solution) via the IV or IO route. This dose should be repeated every 3 to 5 minutes. Alternatively, a one-time dose of vasopressin (40 units) can be given to replace the first or second dose of epinephrine, but not both. Do NOT hyperventilate the patient as doing so increases intrathoracic pressure and can impair venous return (preload) and cardiac output, which would decrease the effectiveness of chest compressions. After an advanced airway has been placed during cardiac arrest, deliver one breath every 6 to 8 seconds (8 to 10 breaths/min) and ensure that chest compressions are uninterrupted. There is presently no evidence to support the efficacy of transcutaneous cardiac pacing (TCP) in patients with bradycardic PEA or asystole. You and your team are performing CPR on a 70-year-old male. The cardiac monitor reveals a slow, organized rhythm. His wife tells you that he goes to dialysis every day, but has missed his last three treatments. She also tells you that he has high blood pressure, hyperthyroidism, and has had several cardiac bypass surgeries. Based on the patient's medical history, which of the following conditions is the MOST likely underlying cause of his condition? A: Coronary thrombus B: Drug toxicity C: Hyperkalemia D: Hypovolemia - ANS*C: Hyperkalemia* Reason: Although any of the listed conditions could be causing this patient's condition, the fact that he missed his last three dialysis treatments should make you most suspicious for hyperkalemia. Dialysis filters metabolic waste products from the blood in patients with renal insufficiency or failure. If the patient is not dialyzed, these waste products, including potassium and other electrolytes, accumulate to toxic levels in the blood. In addition to performing high-quality CPR, managing the airway, and administering epinephrine, your protocols may call for the administration of calcium chloride and sodium bicarbonate if hyperkalemia is suspected. Albuterol also has been shown to be effective in treating patients with hyperkalemia becauses it causes potassium to shift back into the cells; it can be nebulized down the ET tube or administered intravenously. Follow your local protocols regarding the treatment for suspected hyperkalemia.
Docsity logo



Copyright © 2024 Ladybird Srl - Via Leonardo da Vinci 16, 10126, Torino, Italy - VAT 10816460017 - All rights reserved